You are on page 1of 34

FERTILIZATION, IMPLANTATION AND Untuk tahapan proses oogenesisnya direview sendiri

yah, fyi untuk jumlah oosit ini ga bisa nambah jadi


PLACENTATION biasanya dari bayi sampe dewasa ya udah segitu aj
dr. Dicky M Rizal, Sp.And, AIFM, M.Kes b. Sperma atau sel mani, memiliki bentuk yang mirip
Pemateri: Ajeng | Editor: Mala kecebong yang terdiri dari kepala , leher, dan ekor
yang dapat dilihat pada slide ke.6. Nah ekor sendiri
Halo semua, assalamualaikum ^_^. Ketemu lagi dengan saya nih,
kali ini editornya ganti yah. Semoga makin berkulitas HSC-nya. memiliki fungsi sebagai alat gerak dari sperma, yang
Jangan lupa berdoa sebelum baca. Nah kita bahas yah, apa sih getarannya sangat cepat , serta ekor ini memiliki
yang mau kita pelajari dikuliah ini. Jadi materi kuliah in menurut panjang kira-kira 10x dari kepala.
saya semacam overview dari week satu kita, nah disini baiknya Secara embriologi, spermatogonium awal mulanya
kita sambil inget materi praktikum anat dan juga P ya, dan biar adalah sel-sel primitive tubulus testis. Dan jumlah sel
konek juga sambil buka materi/ppt infertilitas dr. Dicky ya hehe sperma ini tidak akan bertambah dari bayi setelah
dilahirkan, namun setelah pubertas akan mengalami
Pasti dah ngerti banget nih apa itu fertilisasi dan juga penigkatan jumlah produksi yang dipengaruhi oleh
implantasi kan. Nah, sekarang kita akan belajar dari sisi sel-sel interstisial Leydig hingga mengalami
fisiologinya. Tak kenal maka tak sayang, jadi nanti kita spermatogenesis.
bakal bahas definisi serta apa aja yang terjadi dari awal
sperma ketemu ovum a.k.a fertlisasi sampe dia akan
nempel di rahim a.k,a implantasi. Dan materi plasentasinya
akan dibahas dikuliah ya

FERTILISASI
Fertilisasi adalah proses bertemunya ovum dan sperma.
Dan hanya sperma yang strong, punya skala % motilitas
dalam analisa sperma yang dapat membuahi sebuah ovum.
Peristiwa fertilisasi ini disebut juga sebagai konsepsi,
dimana pada sel telur terdapat banyak sperma, lalu sperma
yang berkumpul akan mengeluarkan ragi yang berfungsi
mencairkan zat-zat yang melindungi ovum, setelah itu
sperma terbaik akan masuk ke ovum dan bersatu dengan
ovum.
Pada saat pembuahan ini, kepala sperma yang
mengalami kapasitas dapat melewati zona pelusida dan
menembus ke vitelus ovum, serta setelah itu akan ada ASK: apakah ejakulasi itu penting?
perubahan strukstur zona pelusida untuk mencegah ANSWER: ya, untuk fasilitasi sperma saat bertemu dinding
sperma lain masuk (BLOCK TO POLYSPERMIA, kecuali kalau vagina, terutama pada lendir vagina -> mengubah
dinding pelusida jelek). Setelah itu, ada proses penyatuan keasaman vagina dan fasilitasi kehidupan sperma. Fungsi
pronuclei pria dan wanita (syngami) dan terbentuk zigot. yang kedua untuk DECAPITATION FACTOR.
Nah sebelum kita bahas fertilisasi kita juga harus kenal Nah ada beberapa info, cek lagi kuliah male infertility.
dulu apa aja komponen yang berperan di fertilisasi, yaps Kalau sperma yang di epididymis pasti udah matur,
benar. Dia itu ovum dan spermatozoa. Kita akan bahas dlu sehingga sperma yang belum melewati epididymis
tentang ovum yah, ini aku masukin dari buku prof.ova bisa walaupun jumlahnya cukup tapi kemungkinan untuk
dibaca diperpus. membuahi masih cukup kecil karena melewatkan banyak
a. Ovum, berasal dari genital ridge dimana pada wanita proses maturasi.
jumlah ovum tergantung dari umurnya yaitu: Di epididymis, spermatozoa akan.
Baru lahir : 750.000 1. Memperoleh sifat motilitasnya
Usia 6-15 th : 439.000 2. Lose of cytoplastic droplet
Usia 16-25 th : 159.000 3. Mengalami kondensasi akhir dari kromatin. The sperm
Usia 26-35 th : 59.000 surface undergoes various molecular alterations.
Usia 35-45 th : 34.000 4. Yang paling penting Adsoprsi protein (green) yang
Masa menopause : semua hilang diikuti oleh sperm-zona binding dan faktor stabilitas.
Pada saat ejakulasi, cairan sperma akan
bercampur dgn plasma seminal dari glandula
aksesorius. Seminal plasma glycoproteins
(orange) melekat kuat pada permukaan sperma
dan secara efisien menstabilkan sel selama
perjalanan di bagian bawah traktus genitalia
wanita.
Sperma, harus melewati vagina (NATURAL FERTILITY),
ada SERVIKS HOSPITALITY sebab serviks yang baik
akan mentrigger sperma untuk KAPASITASI ketika
sperma mencapai traktus genitalia wanita ismus dari
oviduk. Misal ada gangguan, atau servik gak ramah
misalnya ada leukorhea atau keputihan dapat
menggangu proses kapasitasi yang berakibat pada
kegagalan fertilisasi. Meskipun peristiwa migrasi dari
sperma masih belum diketahui, tapi ada dugaan
bahwa sperma yang melewati uterus atau uterotubal
junction menyebabkan hilangnya faktor dekapasitasi
dan spermatozoa itu kini dapat merespon sinyal
stimulatori yang dikeluarkan oleh ovum ataupun
traktus wanita.

ASK: apakah orgasme penting? Saat orgasme ini aka nada


kontraksi dinding uterus dan tuba.
ANSWER: tidak penting untuk kehamlan, sebab apabila saat
orgasme kanalis servikalis terbuka paling lebar namun tetap
tdk menjamin sperma akan masuk dengan mudah, masih
ada banyak faktor lainnya yang akan berpengaruh pada
kehamilan., serta adanya kontraksi dinding uterus dan tuba
juga tidak akan mempercepat sperma untuk masuk. Tetapi
secara hormonal ada pengaturan tertentu yang akan
mempengaruhi mudahnya sperma untuk dapat masuk ke
dalam kanalis servikalis. Sehingga kontraksi itu tadi cuman
rumor.
Nah hal-hal yang dapat membantu proses masuknya
sperma itu:
1. Adanya kemampuan dari sperma untuk dapat
melakukan perjalanan
2. Kapasitasi yang diperantarai oleh cairan
3. Tanpa orgasme pun secara hormonal isthmus ( bagian
tersempit) akan dapat otomatis membuka
Nah untuk sifat-sifat spermatozoa lainnya bisa dicek di slide
14- 15, dan berikut ini slide 16 yg akan dibahas dikiti.
Lalu apa yang terjadi ketika sperma mencapai zona
pellucida?
1. Attachment dia akan nempel dlu
2. Binding habis itu dia akan mengikat kuat
3. Lalu terjadilah reaksi akrosom (pelepasan enzim)
4. Dan terjadilah penetrasi zona pellucida oleh sperma.

Nice to know ya gaes ;)


Wassermann et al. nih menemukan kalo ternyata ketika
sperma terikat pada ZP3 menyebabkan perubahan aliran
Ca+2 dan Na+ pada plasmalemma sperma yang akan
menyebabkan reaksi akrosom. (ZP1 dan ZP2 tidak
menyebabkan hal ini terjadi).
Sperma terikat pada o-linked oligosaccharide yg
merupakan bagian ZP3 (komponen karbohidrat dari
glikoprotein ZP3).
Reseptor pada plasmalemma sperma yang mengikat
oligosakarida itu masih belum dapat dipastikan, namun
ada yang mengusulkan bahwa bagian PH-20 dari
reseptor-lah yang megikat walaupun penelitian terbaru
blm menyetujui.
Yang terjadi saat ovum dan sperma bertemu:
1. Nukleus sperma akan masuk ke sitoplasma ovum dan
menjadi pronukleus laki-laki
2. Hasilnya, akan terjadi fusi sperma dengan plasmalemma
ovum, lalu nucleus oosit yang sedang berada pada
metaphase meiosis sekunder akan menyelesaikan
pembelahannya dan menghasilkan badan polar yang
lain.
3. Setelah itu, nucleus ovum akan menjadi pronukleus
perempuan.
4. Pronukleus laki laki dan perempuan yang masih haploid
akan bergerak menuju satu sama lain dan saling
Fungsi dari kapasitasi sperma adalah:
melengkapi dan fusi menjadi nucleus zigot yang diploid
Peningkatan kecepatan metabolisme
#eaaa
Flagela/ekor bergerak sangat cepat, yang menghasilkan
5. Zigot kemudian akan mulai membelah
motilitas sperma
Perubahan pada protein plasmalemma menyebabkan Kita bahas lagi yang block to polyspermia, apasih? Intinya
sperma-egg binding dan terjadinya reaksi akrosom akan ada mekanisme dimana sperma lain dihalangi supaya
PRO-Acrosin(Inaktif) akan diubah menjadi akrosin(aktif) ga masuk ke ovum yang sudah dibuahi oleh sperma,
Sperma memiliki kemampuan untuk kemotaksis dengan mekanisme sebagai berikut:
Adanya peningkatan amplitude dari ekor, gerakannya Dilusi
sangat cepat / straight forward Sekresi seperti fertilizin menyebabkan aglutinasi pada
Amplitudo lateral head: kepala dari spermatozoa saat sperma beberapa spesies invertebrate (Tidak terjadi
bergerak harus berada ditengah. Apabila sudut dari pada mamalia)
amplitude lebar maka kualitas dari sperma buruk dan Pengerasan dari membrane vitelina atau zona
sulit membuahi pellucida yang terjadi akibat rupturnya granula cortical
Kibasan dari ekor, apabila seperti orang kesurupan yg (slow block thd polyspermia sekitar 1 hingga 5 menit)
bergerak tdk teratur maka akan mudah menembus
ovum (wkwk ed.)
Depolarisasi dari oolemma perubahan potensial IMPLANTASI (kurang lebih ada 8 hari, hari ke 6 hingga 14)
membrane dari negative ke positif, sehingga sperma Implantasi a.k.a nidasi adalah suatu proses masuknya
tidak dapat fusi dengan oolemma yang bermuatan atau menempel dan tertanamnya hasil dari konsepsi di
positif (fast block thd polyspermia sekitar 1-2 detik). dalam endometrium. Ketika blastula sampai di rongga
Hal ini terjadi akibat influx ion natrium. rahim, jaringan endometrium berada di masa sekresi.
Jaringan endometrium in akan menghasilkan banyak
Ada tujuh kejadian yang menjadi ciri khas telah terjadi desidua, yaitu sel-sel besar yang mengandung banyak
fertilisasi: glikogen serta mudah dihancurkan oleh trofoblast. Hal
Pelepasan Ca++ yang tersimpan dalam reticulum tersebut menyebabkan pada saat nidasi atau implantasi
endoplasma hal ini sepertinya merupakan langkah sering terjadi perdarahan akibat adanya luka desidua
kritis dalam proses fertilisasi (tanda Hartman). Umumnya nidasi ini terjadi di dinding
Reaksi kortikal rupture/pecahnya granula kortikal depan atau belakang (korpus) dekat dengan fundus uteri.
yang terjadi bersamaan dengan pelepasan Ca++. Isi Setelah nidasi, sel-sel dari blastula akan mulai
dari granul yang dilepaskan ke spasium perivitellina berdiferensiasi. Sel-sel yang lebih dekat dengan dinding
akan menyebabkan pengerasan membrane vitelline endometrium membentuk kumpulan bernama inner cell
atau zona pellucida. mast/embryoblast yang akan membentuk diskus embrionik
Aliran masuk Na+ ke dalam sitoplasma telur yang dan kantung kuning telur (yolk sac), sedangkan sel-sel yang
menyebabkan perubahan pada potensial membrane - lebih pipih dan superficial membentuk outer cell mast yang
fast block to polyspermy. akan menjadi trofoblas dan akan berdiferensiasi menjadi
Reorganisasi sitoplasma telur. dua sel, syncytiotropoblast dan cytotrophoblast. Sel-sel dari
Selesainya meiosis ovum. extraembryonal mesoderm yang tumbuh disekitar embrio
Efflux/Aliran keluar ion H+ (hydrogen) menyebabkan akan melapisi bagian dalam trofoblas, dan keduanya
meningkatnya pH sitoplasma hal ini menyebabkan membentuk sekat korionik (chorionic membrane) yang akan
aktifnya previously inhibited synthetic pathways. berkembang dan bergabung dengan amnion menjadi
Meningkatnya metabolisme zigot bersiap untuk membrane amniokorionik.
pertumbuhan. Vili korioles akan berhubungan dengan desidua basal
yang tumbuh bercabang-cabang disebut sebagai korion
frondosum. Sedangkan vili korioles yang berhubungan
dengan desidua kapsularis akan sedikit mendapatkan
makanan dan tergencet kemudian hilang/menipis sehingga
disebut chorion leave (smooth chorion). Pada saat
implantasi ini HCG (human Chorionic Gonadotropin) sudah
diproduksi oleh trofoblas.

Setelah adanya aktivasi telur aka egg actvation


a. Terjadi replikasi DNA berdasarka pronuklei wanita
atau pria
b. Pronuklei wanita dan pria akan bergabung,
c. Persiapan untuk implantasi pertama atau first
cleavage.
Cek gambar slide 40-42 ya
Oke kita bahas lagi ya, apabila beberapa jam sudah
berlalu dari pembuahan lalu terjadi proses pembelahan
zigot yang terjadi selama tiga hari hingga stadium morula.
Hasil dari konsepsi tersebut dapat bergerak sebelum
implantasi yang dipengaruhi oleh arus getaran silia dan
kontraksi dari tuba yang selanjutnya sampai di kavum uteri
akan berbentuk blastula.
HYPEREMESIS GRAVIDARUM FAKTOR RESIKO yang berhubungan dengan hyperemesis
gravidarum:
dr. Edi Patmini SS, Sp.OG
Pemateri: Izah | Editor: Elida
Assalamualaikuuum Berdoa dulu jangan lupa ehe

Hyperemesis gravidarum berarti muntah berlebih pada


ibu hamil. Untuk mual dan muntah biasa disebut dengan
nausea and vomiting during pregnancy (NVP) atau emesis
gravidarum. Emesis gravidarum tidak sama dengan
morning sickness karena mual muntahnya tidak hanya
waktu malam hari. Menurut data, hanya 20% yang
merasakan mual muntah di pagi, sisanya 80% merasakan
mual muntah sepanjang hari.
Mual muntah ini biasanya akan hilang sendiri dan
mencapai puncaknya pada minggu gestasi ke 9. Pada
minggu ke 20, gejala biasanya hilang namun 20% masih
merasakanya sepanjang kehamilan.
70-85% kehamilan selalu disertai dengan gejala mual ETIOLOGI DAN PATOLOGI
muntah. Dengan 67% hanya mengalami mual saja dan Etiologi tidak diketahui
sisanya mengalami mual dan muntah. Meskipun begitu, Etiologi HG masih belum diketahui secara pasti. Tapi
hanya sebagian kecil saja yang mengalami hypermesis kemungkinan ada faktor biologi, fisiologis, psikologis,
gravidarum atau mual muntah berlebih saat kehamilan. dan sosiokultural yang berkontribusi pada HG. Ada juga
Secara statisktik, HG atau hypermesis gravidarum teori lain yang menyatakan mungkin ada adapatasi
insidensinya hanya 0,5-2% dari kelahiran hidup. evolusioner untuk mencegah intake makanan beracun,
Nah, sebenernya kapan sih mual mintah pada nah bentuk adaptasi ini adalah mual muntah biar nggak
kehamilan bisa disebut hyperemesis gravidarum? Yaitu makan sembarangan. Anyway, mual muntah ini kan
ketika: biasanya dirasakan di trimester pertama, dan di
Mengalami episode muntah lebih dari 3x dalam sehari trimester pertama itu kan masa krusial buat neurulasi,
disertai ketonuria organogenesis, dkk. Kesalahan-kesalahan saat masa
Mengalami penurunan berat badan 5% atau > 3 kg. trimester pertama ini bisa berakibat fatal pada janin.
Human chorionic gonadotrophin (HCG)
DIAGNOSIS BANDING HCG adalah faktor endokrin yang paling mungkin
Jadi nggak semua mual muntah saat kehamilan bisa untuk kasus HG. Hal ini didukung oleh insidensi
dikategorikan sebagai hyperemesis gravidarum. Berikut hyperemesis yang mencapai puncaknya pada kehamilan
adalah differential diagnosis atau kemungkinan diagnosis usia sekitar 9 minggu kehamilan, yang mana hampir
lain pada mual muntah. bersamaan dengan puncak kadar HCG. Kadar puncak
HCG tercapai sekitar minggu ke 9, ada sumber jurnal
juga yang bilang puncak HCG sekitar minggu ke 10.
Meskipun begitu, belum ada bukti yang cukup untuk
mendukung hipotesis ini.
Faktor Hormonal
Estrogens: meningkatnya kadar estrogen dan estradiol
Progesteron: meningkatnya kadar ptogesteron
Hipertiroidisme: secara fisiologis, kadar tiroid akan
berubah saat hamil, termasuk berubah karena ada
stimulasi dari HCG
Transient Hypertyroidism of HG
Adrenocrticotrophic hormone (ACTH)
Cortisol, growth hormon, Prolactine
Infeksi H. pylori: infeski kronis H. pylori juga dapat
menyebabkan HG
Psychosomatic / Psikosomatik
Sebenernya biasa aja, tapi psikologisnya bikin sakit.
Contohnya ada kasus ibu hamil yang kalo di depan
suaminya muntah-muntah terus, tapi kalo suaminya
pergi biasa aja.
Secara ringkas etilogi dan patologi HG bisa dilihat pada
diagram ini, kalo nggak keliatan liat di slide ke 10 yak,
ehehe.
STRATEGI PENGOBATAN
Tujuan Terapi
meminimalisir rasa tidak nyaman dan gejala penyakit
mencegah & meminimalisir dehidrasi serta
ketidakseimbangan elektrolit
mencegah & meminimalisir ketonuria
intake cairan baik
mencegah perawatan rumah sakit yang tidak perlu

Manajemen Awal
a. Makan dan minum sedikit tapi sering untuk mencegah
overdistensi dan memastikan pengosongan lambung
sempurna
b. Pada NVP mild to moderate utamakan karbohidrat,
rendah lemak dan asam seperti snack ringan, produk
susu, kacang-kacangan, biscuit kering dan asin, roti,
sereal, crackers, pasta, dan nasi.
c. Makanan protein-predominant: daging, ayam, ikan,
telur
d. Minuman pengganti eletrolit, suplemen nutrisi oral
e. Dukungan emosional, psychosomatic care
TAMPAKAN KLINIS DAN RIWAYAT PENYAKIT f. Wanita dengan mild NVP harus ditangani dengan
Biasanya simptom tidak spesifik. Hal ini penting untuk antiemetics
mengeksklusi kasus langka pada mual dan muntah g. Perawatan dengan ambulans harus digunakan untuk
Temuan klinis: dehidrasi, berat badan berkurang, pasien yang sesuai ketika pelayanan primer tidak
asidosis, dan alkalosis mampu menangani dan PUQE score kurang dari 13
Ada 2 grade keparahan NVP h. Opname di rumah sakit harus dipertimbangkan ketika
Grade 1 : mual muntah tanpa metabolic imbalance terdapat minimal satu dari tanda-tanda berikut:
Grade 2 : simptom sakit yang jelas disertai metabolic mual muntah terus-menerus dan tidak mampu minum
imbalance antiemetic oral
Tingkat keparahan NVP juga diklasifikasikan oleh indeks mual muntah berhubungan dengan ketonuria
objektif dan tervalidasi untuk mual dan muntah seperti dan/atau penurunan berat badan lebih dari 5% tanpa
PUQE score (Pregnancy Unique Quantification of Emesis) pemberian antiemetics
terkonfirmasi/dicurigai menderita penyakit lain
seperti urinary tract infection dan ketidakmampuan
mentoleransi antibiotic oral

Antiemetics
Ada data efficacy dan keamanan untuk antiemetics lini
pertama seperti antihistamin (antagonis reseptor H1)
dan phenotiazines. Harus diresepkan jika dibutuhkan
untuk NVP dan HG.
Kombinasi obat harus digunakan pada wanita yang tidak
berespon pada antiemetics tunggal.
Untuk wanita dengan HG persisten dan berat,
penggunaan antiemetic pemberian obat rectal atau
parenteral mungkin dibutuhkan dan lebih efektif
dibandingkan dengan oral regimen.
Pasien harus ditanyakan tentang riwayat reaksi pada NVP berat merupakan penyebab terbanyak ketiga
terapi antimeteics sebelumnya. wanita hamil opname di RS
Drug-induced extrapyramidal symptoms dan oculogyric Setiap tahunnya ada 8,5 juta lost working days akibat
crises dapat terjadi pada penggunaan phenothiazines NVP
dan metoclopramide. Jika hal ini terjadi, terapi harus
dihentikan segera. KOMPLIKASI SERIUS
Dokter harus menggunakan antiemetics yang familiar Mallory-Weiss syndrome
dan harus menggunakan obat dari kelas lain juka obat Esophageal rupture
pertama tidak efektif. Pneumothorax
Metoclopramide aman dan efektif, namun karena resiko Peripheral neuropathy
efek extrapyramidal, metoclopramide digunakan Coagulopathy
sebagai obat lini kedua. Wernicke's encephalopathy
Ada evidence bahwa ondansetron itu aman dan efektif, Pre-eclampsia
namun karena data yang ada terbatas, ondansetron Fetal growth retardation
menjadi obat lini kedua.
Rekomendasi terapi dan dosis antiemetics Terimakasih, selamat belajar coooy
Bisa buka referensi ttg hyperemesis gravidarum pada William's
Obstetrics (210, 1050-1052) atau William's Gynecology (140, 781).

INTRODUCTION TO CONTRACEPTION
dr. Agung Dewanto, Sp. OG (K), Ph.D
Pemateri: Berian | Editor: Geegee
Assalamualaikum wr.wb semua!! Gak kerasa ya kita sekarang
udah di blok C1, sudah tahun ke-3, waktu berjalan dengan cepat
ya guys :3. Di sini aku akan mencoba untuk share mengenai
Terapi lainya
kontrasepsi ya... sebelumnya mohon maap karena aku baru bisa
Pyridoxine: tidak direkomendasikan pada NVP dan HG ngeluarin HSC nya di Week 3 karena menunggu slide dari
Corticosteroids: hanya digunakan pada kasus dimana dokternya... Yuk kita samasama bahas mengenai kontrasepsi ini :3
terapi standard gagal
Diazepam: tidak direkomendasikan untuk manajemen INTRODUCTION
NVP ataupun HG Perencanaan keluarga merupakan suatu hal yang sangat
Rehidrasi penting pada sekarang ini. Program pemerintah yang
Normal saline dengan tambahan potassium chloride menggunakan program KB merupakan salah satu bentuk
dengan pemberian yang dipandu oleh monitoring perencanan keluarga yang dilakukan di Indonesia. Lalu
elektrolit harian. Yang paling tepat adalah hidrasi mengapa sih perlu ada perencanaan keluarga ini? kita lihat
intravena. kembali bahwa semua orang pasti ingin memiliki penerus
Infus dextrose kurang tepat kecuali jika kadar serum yang berkualitas. Sehingga perlu adanya perencanan mulai
sodium normal dan thiamine telah diadministrasikan. dari prenatal care hingga lahir dan memiliki anak
Complimentary therapy selanjutnya. Kontrasepsi ini berperan dalam menunda
Ginger / jahe: dapat digunakan untuk antiemesis pada kehamilan, melindungi kondisi kesehatan ibu yang akan
NVP mild to moderate sangat turun apabila hamil (keganasan, heart disease,
Acustimulations: acupressure dan akupuntur aman diabetes), dan lainnya .
untuk kehamilan dan dapat memperbaiki NVP Okee sudah cukup introduction dari aku, sekarang kita
Hypnosis: tidak direkomendasikan untuk NVP dan HG masuk ke materi hscnya yaa

Discharge instructions untuk pasien NVP DEFINISI


Kontrasepsi adalah metode atau alat yang digunakan untuk
mencegah kehamilan

KONSELING
Saat pasien datang ke kita untuk menanyakan mengenai
kontrasepsi ini maka kita sebagai dokter umum harus dapat
menjelaskan dan membantu memilihkan alat kontrasepsi
yang tepat. Ingat pada konseling ini kita bukan yang
memilihkan tapi kita harus melakukan shared decision
making jadi kita praktekkan ya BCCT nya :3 . Kemudian
PREGNANCY OUTCOME AND PROGNOSIS dalam memilih kontrasepsi yang tepat maka kita akan
Pada umumnya NVP akan sembuh sendiri dan hilang merujuk kepada WHO Pearl Index dan Medical Eligibility
pada usia gestasi 20 minggu Criteria. Untuk yang WHO Pearl Index akan aku jelaskan di
NVP dan HG dapat menyebabkan kerugian produktivitas sub bagian klasifikasi ya :3. Selain itu perlu diingat bahwa
baik langsung maupun tidak langsung penggunaan kontrasepsi di Indonesia masih banyak
diperdebatkan sehingga kita sebagai dokter harus bisa hormonal? Untuk mengetahuinya kita akan merujuk pada
menjelaskan dengan baik sehingga tidak sampai melanggar pearl index. Hah? Apa sih itu? yuk mangga dibaca
etika dan norma yang ada. bawahnya yaa wkwk

Medical Eligibility Criteria Pearl Index


Nah waktu pasien datang konsul mau pake KB, kita Adalah angka kejadian kehamilan per 100 wanita per
kudu buka chart medical eligibility criteria ini buat tahun.
menilai boleh apa engga ni pasien dikasih kontrasepsi Terus fungsinya apa sih? Buat nentuin efikasi atau
Siapa yang bikin? Kalau ini guideline standarnya dari keefektifitasan metode kontrasepsi yang digunakan
WHO, tetapi beberapa negara ada yang memodifikasi Faktor-faktor yang memengaruhi kehamilan antara lain
dan membuat sendiri. adalah fertilitas dari pasangan tersebut, waktu
Isi dari chart ini tuh penilaian kondisi-kondisi medis berhubungan seksual dengan ovulasinya, metode
yang dikategorikan dari 1 sampai 4 kontrasepsi, faktor intrinsik keefektifitasan metode
Nanti skornya akan dijumlah dan dikategorikan lagi di tersebut, dan cara penggunaan yang tepat dari
interpretasi metodenya.
Interpretasinya ? Ini ada tabelnya, yang bisa jadi trigger dulu sebelum
kita bahas satu per satu :3
Kategori Interpretasi
1 Boleh memakai kontrasepsi, tidak ada risiko
2 Keuntungan dari penggunaan kontrasepsi >
risiko yang mungkin timbul
3 Risiko yang mungkin timbul > keuntungan dari
penggunaannya
4 Tidak boleh memakai kontrasepsi

Terus kalau mau liat list lengkapnya dimana? di slide


sama dokternya ditulis see pdf file wkwkw Untuk
quick reference chartnya ini nanti kalian bisa lihat di
dropbox ada kok kalau mau jelasnya, tapi ini aku
masukkin juga gambarnya cm biar tau yaa tabelnya yg
ini tp maap ga bs dibaca. Kalau mau liat yang
lengkapnya bisa di sini
http://www.who.int/reproductivehealth/publications/f
amily_planning/Ex-Summ-MEC-5/en/
Ini jgn lupa dipelajarin ya guys!
METODE KONTRASEPSI HORMONAL
KONTRASEPSI ORAL
Kontrasepsi dalam bentuk oral (pill) terbagi menjadi dua
jenis. Ada yang Progestion-only Contraceptive group dan
ada yang Combined Hormonal Contraceptive. Sesuai
namanya yang progestin only isinya progestin aja tapi yang
combined biasanya gabungan beberapa hormon.
Estrogen dan progestin bekerja secara sinergis untuk
menginhibisi ovulasi, menghambat motilitas sperma,
dan mencegah fertilisasi
Dosis yang diberikan pada kontrasepsi oral awalnya 10
mg tapi nanti akan diturunkan menjadi 0,5 1 mg
Bentuk sediaannya biasanya berupa kombinasi
Combined Oral Contraceptive (COC) , biasanya
dosisnya diberikan secara monophasic (dari awal
Kondisi-kondisi yang dapat meningkatkan risiko pemberian sampai akhir dosisnya selalu tepat ).
kehamilan yang tidak terencana: Kandungannya adalah 30 mcg EE (Ethynil Estradiol)
1. Kanker payudara atau Estrogen Dependent dengan noresthisterone atau levonogestrol. Selain
Neoplasia dosis monophasic, masih ada juga dosis berupa
2. Complicated Vascular Heart Disease biphasic(berubah 2x), triphasic(3x), dan
3. Diabetes : Vascular complications quadriphasic(4x). Pemberian COC pada pasien dengan
thrombus tidak diperbolehkan karena COC ini
KLASIFIKASI meningkatkan risiko VTE (Venous Thrombo Embolism).
Kontrasepsi dibagi menjadi dua yaitu hormonal dan non Selain itu sifatnya lebih estrogenic.
hormonal. Nah lebih efektif mana hormonal atau non- Terus mekanisme aksi dari COC ini gimana sih?
Jadi dia akan menekan hypothalamic gonadotropin Neoplasia dapat menyebabkan hepatic focal
releasing factors akibatnya akan memblok sekresi FSH nodular hyperplasia dan benign hepatic adenoma
dan LH oleh pituitari. Komponen progestin akan
mencegah ovulasi melalui suppresi LH; menebalkan TRANSVAGINAL RING : NUVARING
mukus serviks dan menghambat laju sperma.
Sementara itu estrogen akan memblok ovulasi dengan
menekan pelepasan FSH, selain itu akan menstabilisasi
endometrium yang menghambat perdarahan
intermenstrual.
Aksi farmakologis Progestin dan Estrogen
Progestin Estrogen
Ovarian and pituitary Ovarian and pituitary Salah satu pilihan metode kontrasepsi yang banyak
inhibition inhibition digunakan
Penebalan mukus serviks Penipisan atau peningkatan Cara pakenya adalah dengan dimasukkan ke dalam
mukus serviks vagina tanpa ada posisi intravaginal khusus. Kemudian
Atrofi endometrium Proliferasi endomterium setelah masuk maka NuvaRing ini akan melepaskan
ethinyl estradiol dan progestin etonogestrel yang akan
Cycle control Cycle control
diabsorbsi oleh epithellium vagina.
Pemasangannya adalah dalam 5 hari sejak onset
Setiap tablet kontrasepsi oral isinya berbeda-beda
menstruasi dan akan dipakai terus selama 3 minggu.
tergantung kebutuhan
Setelah itu dilepas selama 1 minggu untuk membiarkan
Pada prinsipnya, kontrasepsi oral ini akan disesuaikan
darah keluar.
dengan fase menstruasi wanita. Sehingga prinsip
Jika dilepaskan >3 jam, maka gunakan metode back up
kerjanya menyerupai siklus menstruasi cuman bedanya
selama 7 hari
dia kadarnya lebih tinggi dari normal karena dia akan
Tingkat laju kehamilannya adalah 0,65 per 100 wanita
menginhibisi yang ada di sentral (pituitary)
per tahun. Kemudian tingkat kepuasannya cukup
Penggunaan estrogen sendiri harus hati-hati karena jika
tinggi, tetapi sering dijumpai vaginitis, ring-related
berlebihan bisa menjadi kanker , progesteron juga akan
events ( seperti terasa saat berhubungan seksual ), dan
menekan sistem imun sehingga jika kelamaan dapat
leukorrhea
menyebabkan ibunya vulnerable terhadap penyakit.
Progestin only contraceptive group isinya progestin aja
TRANSDERMAL PATCH : ORTHO EVRA
terus dia membuat endometrium jadi enggak siap dan
mukosa serviksnya jadi ga ramah sperma. Selain itu
kerugian dari grup ini adalah biasanya dia ada irregular
uterine bleeding
Permasalahan utama dari pil kontrasepsi adalah kepatuhan
dari si pasien untuk meminum pil tersebut karena banyak
sekali kasus dimana si Ibu lupa minum kemudian menjadi
hamil sehingga kontrasepsinya tidak berjalan
Keuntungan dari kontrasepsi oral

Penggunaannya adalah dapat ditempelkan pada


pantat, lengan atas luar, perut bawah, atau upper
torso, tapi payudara harus dihindari
Fyi ortho evra ini water resistance jadi bisa di pake
buat berenang ataupun yang lain tanpa perlu
melepasnya
Isinya 6 mg norelgestromin dan 0,75 mg ethinyl
estradiol
Method specific effects Penggunaannya adalah 3 minggu dan setelah itu harus
Mengganggu efikasi beberapa obat, contohnya dilepas selama 1 minggu, jadi mirip miriplah soalnya
acetaminophen biar darah mensnya keluar. Terus inget, karena ini
Perubahan metabolik COC meningkatkan kadar patch atau bahasa awamnya koyo maka kita harus
serum trigliserida dan total kolesterol masangnya yang bener kalau ga bener nanti efeknya ga
Efek kardiovaskulat meningkatkan risiko Deep Vein maksimal apalagi kalau misalkan ternyata dia ga
Thrombosis dan emboli pulmo nempel maka harus diganti ya...
Terus kerugiannya ada reaksi lokal, ga efektif pada
wanita dengan BB >89 Kg, efek samping sama kayak
kontrasepsi oral tapi lebih meningkatkan risiko nyeri disesuaikan dengan axis panjang tangan. Mekanisme
payudara dan juga dysmenorrhea dan yang terakhir ga aksinya hampir sama lah ya yaitu menekan ovulasi.
ada perlindungan terhadap HIV atau STI Kemudian implan ini cocok untuk ibu yang post partum > 4
minggu
KONTRASEPSI DARURAT
Adalah terapi yang diberikan kepada wanita yang KONTRASEPSI INTRAUTERINE
melakukan hubungan seksual tanpa pengaman,
termasuk juga pemerkosaan
Biasanya dikenal juga dengan morning-after pill
Tatalaksana
o Pemberian kontrasepsi darurat hormonal, bisa
sampai 72 jam setelah onset
PREVEN, Plan B sampai 120 jam setelah
onset, tapi efektivitasnya berkurang 150mg
dari Levo (?? Ini kurang paham guys )
Hampir semua wanita yang ingin kontrasepsi dengan
Copper IUD (up to 5 days after ovulation)
efektifitas tinggi dapat menggunakan IUD. IUD sendiri dapat
Mifepristone (off label, up to 120 hours after
digunakan pada ibu yang belum pernah hamil dan juga
unprotected sex)
yang ada infeksi Upper Genital Tract atau pernah ada
o Dosis yang direkomendasikan oleh WHO :
riwayat kehamilan ektopik. Tetapi kalau ternyata si Ibu
1 dosis levonorgestrel 1.5 mg, atau 1 dosis
sedang menderita STD ( Sexual Transmitted Disease ) maka
ofulipristal 30 mg, diminum dalam 5 hari (120 hari)
jangan dipasang IUD.
dari onset intercourse, atau 2 dosis pil
1. LNG IUS ( Levonorgestrel Intrauterine System )
kontraseptic oral kombinas (regimen Yuzpe)
Dikenal juga dengan nama Mirena. Isinya ada
levonorgestrel yang merupakan derivat dari progestin.
DEPOMEDROXYPROGESTERONE ACETATE ( DMPA )
Efek dari Mirena ini sifatnya sistemik, karena dia akan
Merupakan kontrasepsi hormonal jenis injeksi
melepaskan progestin ke uterus dalam laju yang relatif
Dosis regimen 150 mg setiap 3 bulan sangat efektif konstan dan pada akhirnya menurunkan efek sistemik.
untuk mencegah kehamilan Penggunaannya sendiri adalah sampai 5 tahun. Cara
Alasan paling penting mengapa perempuan berhenti kerjanya si Mirena ini adalah dengan mengubah
menggunakan DMPA adalah adanya perdarahan mukosa serviks sehingga membuat motilitas dan fungsi
vaginal ireguler yang persisten. Selain itu penggunaan sperma terhambat. Gampangnya suasanya jadi gak
DMPA berhubungan dengan peningkatan BB mendukung buat sperma. Selain itu dia juga punya
Mekanisme aksi: Durasi proteksi: 3 bulan (13 mgg) weak foreign body action. Dan yang terakhir karena dia
o Menghambat ovulasi melepaskan progestin dalam jangka panjang maka
Menekan kadar FSH dan LH dapat menyebabkan atrofi endometrium yang
Meniadakan LH surge menyebabkan implantasi normal terhambat. Dosisnya
o Menebalkan mucus serviks 20 mcg/d LNG ( dosis 30 dapat menekan ovulasi )
o Mencegah penetrasi sperma 2. Cu- IUD
o Mengurangi transpor sperma ke tuba fallopi Merupakan jenis IUD yang paling sering kita jumpai
o Atrofi endometrium nantinya. Kita juga udah belajar cara masangnya kan
Contoh : Depo-Provera - 150 mg DMPA dengan guys :3. Penggunaan Cu-IUD ini harus dihindari pada
injeksi IM; Depo-subQ Provera 104 - 104 mg injeksi pasien dengan STD atau PID ( Pelvic Inflammatory
subkutan Disease ). Hampir semua wanita dapat menggunakan
IUD ini tetapi kontraindikasi pada wanita post partum
IMPLANTS atau post abdominal infection dalam 3 bulan terakhir,
uterine or cervical cancer, infeksi serviks, dan alergi.
Cara dia kerja adalah dengan mencegah fertilisasi
dengan menginduksi reaksi inflamasi pada
endometrium. Komponen seluler dan humoral dari
inflamasi tersebut akan diekspresikan pada jaringan
endometrium dan pada cairan yang ada di cavitas
uterine dan tuba fallopi. Sehingga akan menurunkan
viabilitas sperma dan ovum.
Implant termasuk ke dalam Long Acting Reversible Efek samping pemasangan IUD
Contraception. Impant sendiri terbagi menjadi dua yaitu Perforasi pada saat sonde uterine atau pemasangan
etonogestrel implants dan levonogestrel implants. Pada IUD, uterus dapat terperforasi
kuliah ini yang diajarkan kepada kita yang etonogestel Hilang biasanya sering terjadi pada bulan pertama
implant, jadi yang ada disini lebih fokus ke yang sehingga perlu dimonitor dan dicek IUD nya berada
etonogestrol itu yaa. Nama implannya adalah Implanon. dimana
Implan ini akan diletakkan secara subdermal pada bagian Perubahan siklus menstruasi dapat menyebabkan
medial lengan atas 8-10 cm dari siku pada sulkus biceps dan dysmenorrhea dan menorrhagia. Untuk pengguna
Mirena maka harus dikasih tau biasanya ada irregular Fyi cuman ada satu jenis yang ada di pasaran yaitu FC2
spotting sampai 6 bulan pertama Female Condom dan juga pemasangannya ribet
Infeksi pemasangan IUD dapat menyebabkan Pelvic
Inflammatory Disease. Jika infeksinya terjadi pada KONDOM PRIA
bulan pertama, maka hal ini berhubungan dengan Keuntungan idem yang wanita, selain itu murah dan
pemasangan IUD karena sebelumnya sudah ada infeksi banyak di pasaran
tetapi tidak terdeteksi oleh dokter yang memasang Kerugian cuman sekali pakai, bisa robek,
Konseling IUD sudah ada di Shared Decision Making BCCT mengurangi sexual pleasure
ya guys SPERMICIDES
Fungsinya adalah dengan menyediakan physical
METODE BARRIER barriers kepada penetrasi sperma dan aksi spermicidal
FERTILITY AWARENESS BASED METHODS kimiawi
Metode kalender nanti dihitung waktu yang Isinya adalah nonoxynol 9 atau octoxynol 9
tepatnya kapan Dipake 1 jam sebelum berhubungan seksual dan tidak
Basal Body Temperature dilihat peningkatan melindungi terhadap STD atau HIV, dan juga dapat
temperature tubuh dimana biasanya saat tepat mau merusak mukosa vagina atau epitel servis
ovulasi
Metode mukosa serviks ( Billings method ) yang STERILISASI
dilihat kondisi mukusnya apakah kering atau basah, Keuntungan buat yang udah gamau punya anak,
tapi harus abstinence dari awal mens sampe 4 hari cepet sembuh, efek jangka panjang sedikit, cost-
setelah mukus teridentifikasi effective, tidak perlu menggunakan kontrasepsi
Sympto-thermal method gabungin metode basal sebelum berhubungan, derajat keamanan tinggi
body ama mukosa Kerugian permanen, penyesalan terhadap
keputusannya, tidak melindungi terhadap HIV atau STI
DIAFRAGMA lainnya, prosedural yang dibutuhkan cukup kompleks
Komplikasi komplikasi prosedural seperti
perdarahan, infeksi, dll. Selain itu adalah kehamilan
ektopik
Di Indonesia dikenal dengan nama kontrasepsi mantap
( vasektomi dan tubektomi ). Untuk tubektomi sendiri
caranya adalah dengan mengoklusi (mengikat dan
Keuntungan dapat digunakan berjam-jam sebelum memotong atau memasang cincin) tuba fallopi agar
berhubungan seksual, tetapi jika sudah lebih dari 6 jam sperma tidak bisa bertemu dengan ovum
maka perlu diberikan tambahan spermicide pada
vagina atas. Selain itu keuntungannya adalah tidak POST PARTUM CONTRACEPTION
perlu diganti antara coitus METODE AMENORRHEA LAKTASI
Kerugian dapat memicu alergi, ukurannya harus Kontrasepsi yang mengandalkan pemberian ASI secara
sesuai dengan bentuk anatomis sehingga perlu resep, eksklusif, artinya hanya diberikan ASI tanpa tambahan
dibutuhkan insersi dan removal vagina, harus makanan ataupun minuman apa pun lainnya
digunakan minimal 6 jam dan maksimal 24 jam setelah 3 yes:
berhubungan seksual, dan yang terakhir dapat o Are you amenorrheic? berarti ga ada
meningkatkan risiko UTI dan Toxic Shock Syndrome perdarahan menstrual atau perdarahan lain
yang berdurasi selama >2 hari (perdarahan
KONDOM WANITA pada 2 bulan pertama dieksklusi)
o Are you fully or nearly fully breastfeeding?
maksudnya adalah bener2 ngasih bayinya ASI
eksklusif tanpa makanan/minuman lain
o Is your infant less than 6 months of age?
Efikasi LAM (lactation amenorrhea) Subsequently,
studies of method use have consistently found a 6-
month pregnancy rate averaging 2%
Cara kerjanya adalah dengan menunda atau menekan
ovulasi. Jadi saat memberikan ASI akan terjadi
Keuntungan perlindungan terhadap STI, gak perlu peningkatan prolaktin yang menginhibisi sekresi GnRH
resep, bisa dimasukkan sebelum berhubungan, dan dari hipotalamus. Yang pada akhirnya mengganngu
terbuat dari latex free material (polyurethane) HPO-axis, mencegah sekresi estrogen dan ovulasi
Kerugian tidak seefektif kondom pria, harus Metode kontrasepsi hormonal yang dapat dipilih untuk ibu
dimasukkan dan dilepaskan oleh wanita, cuman satu menyusui adalah progestin only contraceptives. Selain itu
ukuran, sekali pakai, berisik, lingkaran luar sedikit IUD juga direkomendasikan sebagai pilihan untuk wanita
mengganggu yang aktivitas seksualnya aktif. Estrogen-Progestin
Contraceptives dapat menurunkan laju dan durasi ASI
sehingga tidak dipilih sebagai metode kontrasepsi untuk ibu e. Development of antibodies
post partum 4. A 30-year-old woman presents for a physical
examination for work. She denies any medical
Alhamdulillah materinya udah selese, di bawah ini ada latihan problems or surgeries in the past. She has had no
soalnya ya guys :3 semoga hscnya bermanfaat bagi kita semua ya pregnancies. She is sexually active and has been using
:D oral contraceptive pills for the past 6 years. She denies
any allergies to medications. On examination, her
LATIHAN SOAL weight is 62 kg, blood pressure 120/78 mm Hg, pulse
1. A 21-year-old woman presents to your office for her 76 beats per minute, respiratory rate 15 breaths per
well-woman examination. She has recently become minute, temperature 36.8C (98.4F). Her physical
sexually active and desires an effective contraceptive examination is normal. Which of the following is a
method. She has no medical problems, but family known benefit of combination estrogen plus progestin
history is significant for breast cancer in a maternal oral contraceptives?
aunt at the age of 42.She is worried about getting a. Decreased activity of rheumatoid arthritis
cancer from taking birth control pills. You discuss with b. Decreased risk of breast cancer
her the risks and benefits of contraceptive pills. You tell c. Decreased risk of cervical dysplasia
her that which of the following neoplasms has been d. Decreased triglyceride levels
associated with the use of oral contraceptives? e. Decreased risk of gallbladder disease
a. Breast cancer 5. A 32-year-old woman presents to your office to discuss
b. Ovarian cancer contraception. She has recently stopped breast-feeding
c. Endometrial cancer her 8-month-old son and wants to stop her progestin-
d. Hepatic cancer only pill because her cycles are irregular on it. You
e. Hepatic adenoma recommend a combination pill to help regulate her
2. A 39-year-old G3P3 presents for her postpartum cycle. You also mention that with estrogen added, the
examination and desires a long-term contraceptive contraceptive efficacy is also higher. In combination
method, but is unsure if she wants sterilization. She birth control pills, which of the following is the primary
has been happily married for 15 years and denies any contraceptive effect of the estrogenic component?
sexually transmitted diseases. Her past medical history a. Conversion of ethinyl estradiol to mestranol
is significant for mild hypertension, for which she takes b. Atrophy of the endometrium
a low-dose diuretic. She is considering the copper c. Suppression of cervical mucus secretion
intrauterine device and wants to know how it works. d. Suppression of luteinizing hormone (LH) secretion
Which of the following is the best explanation for the e. Suppression of follicle-stimulating hormone (FSH)
mechanism of the action of the copper intrauterine secretion
device (IUD)?
a. Decreased tubal motility inhibits ovum transport. Jawaban : EDCAE
b. Chronic bacterial endometritis interferes with Referensi :
implantation. - Berek, J. S. 2012. Berek & Novaks Gynecology 15th
c. Premature endometrial sloughing associated with Edition. LWW: Philadelphia.
menorrhagia causes early abortion. - BKKBN dan Kemenkes RI. 2012. Pedoman Pelayanan
d. An inflammatory response within the endometrium Keluarga Berencana Pasca Persalinan di Fasilitas
kills sperm. Kesehatan.
e. Thickened cervical mucus blocks sperm transport. - Cunningham, F.G., et al. 2014. Williams Obstetrics 24th
3. A 21-year-old G0 presents to your office because her Edition. McGraw Hill: New York
menses is 2 weeks late. She states that she is taking her - Dewanto, A., 2016. Introduction to Contraception.
birth control pills correctly; she may have missed a day - Schnerider,K. M. dan Stephen K. Patrick. 2012. Obstetrics
at the beginning of the pack, but took it as soon at she & Gynecology 13th Edition. McGraw Hill: New York.
remembered. She denies any medical problems, but 3
or 4 weeks ago she had a viral stomach flu and
missed 2 days of work for nausea, vomiting, and
diarrhea. Her cycles are usually regular even without
contraceptive pills. She has been on the pill for 5 years
and recently developed some midcycle bleeding, which
usually lasts about 2 days. She has been sexually active
with the same partner for the past 3 months and has a
history of chlamydia 3 years ago. She has had a total of
10 sexual partners. A urine pregnancy test is positive.
Which of the following is the major cause of unplanned
pregnancies in women using oral contraceptives?
a. Breakthrough ovulation at midcycle
b. High frequency of intercourse
c. Incorrect use of oral contraceptives
d. Gastrointestinal malabsorption
HYDATIDIFORM MOLE Abnormalitas mola terjadi sejak masa fertilisasi. Penelitian
mengatakan bahwa kelainan kromosom berperan penting
dr. Shinta Prawitasari, M.Kes, Sp.OG(K) dalam perkembangan mola hydatid. Dari tampakan
Pemateri: Arin | Editor: KD histologis, perbedaan karyotype, dan ada tidaknya elemen
embryo, Mola dibedakan menjadi 2, yaitu complete dan
Halooo jangan lupa berdoa dulu ya sebelum dan sesudah partial. Berikut penjelasannya,
belajar. Semangat! Mola complete merupakan hasil dari androgenesis,
dimana hasil kromosom haploid (46 XX) berasal dari
INTRO sel paternal (sperma). Ini dapat terjadi karena ovum
Kali ini kita bakal belajar yang namanya Mola Hydatidosa yang kosong atau inaktif dibuahi oleh satu sperma dan
alias Hydatiform Mole alias Molar Pregnancy, tapi kita bakal pada saat meiosis pronukleus sperma sendiri yang
lebih sering sebut Mola gitu aja ya, biar singkat. Mola membelah menjadi haploid (46 XX) ataupun ovum
sempat disinggung di blok B berapaaa aku lupa. Kali ini kita yang kosong atau inaktif dibuahi oleh dua sperma
bakal bahas lebih spesifik ke Mola berikut terapi dan (dyspermic ini lebih jarang) sehingga kromosom yang
manajemennya. Kita mulai dari kasus: dihasilkan bisa 46XY atau 46XX.
Wanita 32 tahun datang ke rumah sakit karena sedikit
perdarahan vagina. Ia hamil 10 minggu. Uterusnya
seukuran uterus usia kehamilan 12 minggu. Tekanan
darahnya 140/90 (hipertensi) dan pulsasi cepat 120
kali/menit (palpitasi). Ia berkeringat walaupun suhu
ruangan hanya sekitar 24 derajat celcius.
Jika kita dihadapkan dengan kasus tersebut, apa diagnosis
banding nya? Tampakan mikroskopis Mola Complete ditandai
1. Mola hydatidosa adanya vili yang membesar dan edema, disertai
2. Kehamilan ektopik proliferasi tropoblastic yang melibatkan seluruh
3. Abortus plasenta. Secara makroskopis, perubahan tersebut
4. Kelainan ginekologis (e.g. cervicitis, cervical polyp, ca membuat villi chorionic akan nampak seperti vesikel-
cervix) vesikel tanpa adanya fetus.
Jika diagnosisnya adalah kehamilan ektopik, maka
umumnya pasien akan datang dengan tiga keluhan utama
yaitu nyeri pelvis, perdarahan dan amenorrhea. Jika
diagnosisnya adalah abortus maka biasanya darah yang
dikeluarkan akan lebih banyak, kecuali pada abortus
iminens. Dan, jika diagnosisnya adalah kelainan ginekologi
seperti cervicitis, cervical polyp dan ca cervix, maka pada
pemeriksaan fisik ginekologi akan ditemukan peradangan
Mola partial memiliki karyotype triploid, bisa 69XXX,
ataupun masa di serviks.
69XXY atau yang lebih jarang 69XYY. Hal ini merupakan
Pada kasus tersebut yang kita soroti adalah usia kehamilan
hasil fusi dari sel ovum dan dua sel sperma (dyspermic
10 minggu, tapi ukuran uterus seperti usia 12 minggu -
fertilization). Kondisi umunya letal untuk fetus, jika
>uterus membesar sebelum waktunya. Juga ada tanda-
pun berkembang dan hidup fetus bisa mengalami
tanda naiknya metaolisme tubuh dengan berkeringat,
gangguan pertumbuhan dan kelainan kongenital.
tekanan darah tinggi dan palpitasi. Apakah tanda ini cocok
dengan diagnosis Mola? Lets check!

DEFINISI
Mola hydatidosa/Hydatidiform Mole/Molar Pregnancy
adalah kehamilan abnormal yang secara histologis Umumnya, Mola parsial mengandung jaringan fetusa
bercirikan kelainan plasenta, khususnya vili chorionic yang dan amnion selain jaringan plasenta (yang
menampakkan bermacam derajat proliferasi tropoblat hiperproliferasi). Biasanya nampakan vesikelnya juga
dengan edema stroma vili. tidak terlalu menonjol. Ini gambarnya
Mola hydatid adalah bagian dari GTD (Gestational
Trophoblastic Disease). GTD adalah istilah yang digunakan
untuk sekelompok tumor yang disebabkan oleh proliferasi
tropoblast yang abnormal. Secara histologis GTD dibagi
menjadi 2; (a) hydatidiform mole yang ditandai dengan
terlibatnya villi, dan (b) nonmolar tropoblastic neoplasm
yang tidak melibatkan villi. Nonmolar tropoblastic
neoplasm sendiri meliputi choriocarcinoma, placental site
tropoblastic tumor, dan epitheloid thropoblastic tumor,
yang dibedakan dari tipe tropoblastnya. Kita ga akan bahas
ini lebih lanjut gengs, utamanya Mola dulu aja.
Perbedaan komplit dan parsial dapat dilihat di tabel berikut

Ukuran uterus lebih besar dari usia kehamilan (yang


dipake usia gestasi). Seperti di contoh kasus, usia
kehamilan ibu 10 minggu tapi ukuran uterusnya sudah
setara usia 12 minggu. Tapi bisa aja ibu datang dengan
ukuran uterus normal.
Kalo di williams, katanya wanita yang hamil Mola
complete bisa didiagnosis pada usia kehamilan sekitar
SIGN & SYMPTOMS 10 minggu.
Mola Complete Mola Incomplete
Pasien Mola complete biasanya datang dengan Pasien Mola parsial memiliki tanda dan gejala mirip
perdarahan vagina (pada 50% kasus). Perdarahan aborsi inkomplit atau missed abortion. Perdarahan
bervariasi dari sedikit hingga banyak yang disertai sekedar spotted bleeding (flek flek)
gelembung vesikel Mola saat dilakukan pemeriksaan
inspekulo. PEMERIKSAAN LANJUTAN
Pasien juga mengalami mual dan muntah yang berlebih Level -hCG Serum
(parah) karena level -hCG yang sangat tinggi. Gejala Kayaknya ini ga lazim dilakukan di Indonesia, di slide ga
hipertiroidisme juga dapat muncul karena stimulasi disebutin, ini kuambil dari Williams. Mungkin karena
glandula tiroid yang disebabkan oleh hCG (menyerupai alasan fasilitas juga. Tapi yang jelas tes kehamilan (PP
TSH) atau juga tyroid stimulating substance yang test) tetap dilakukan, biasanya pake strip test.
diproduksi oleh tropoblast. Namun tanda tanda Walaupun bisa menimbulkan false negatif kalau kadar
hipertiroid ini tidak selalu ada. -hCG terlalu tinggi (karena strip test hanya bisa
Tanda hipertiroidisme yang dapat muncul antara lain: membaca kadar -hCG pada level 20-25mIU/L,
Nadi cepat (palpitasi) sehingga jika kadarnya terlalu tinggi malah tidak
Cemas terbaca), tes ini tetap dilakukan. Jika hasilnya negatif,
Intoleransi terhadap panas (selalu merasa panas jangan langsung percaya, urin bisa diencerkan terlebih
walau suhu ruangan normal) dahulu dan dilakukan tes ulang. Jika kadarnya memang
Turun berat badan tanpa alasan (orangnya kurus tinggi, tes kedua hasilnya positif.
ga sehat) USG
Diare Kebanyakan diagnosis Mola ditemukan setelah USG
Tremor karena bisa ditemukan pembengkakan yang meluas
Kulit basah karena keringat dan perbesaran vili chorionic.
Pre-eklamsia dapat ditemukan pada 2% kasus Mola Mola komplit akan nampak sebagai masa uterus
complete, biasanya kehamilan yang sudah besar. Hal ini ekogenik dengan banyak ruang kista yang anekoik
dapat terjadi karena masa hipoksik tropoblast yang namun tanpa disertai fetus ataupun kantong amnion
melepaskan faktor antiangiogenik yang menyebabkan (disebut snow storm).
jejas endotel. Sedangkan Mola parsial nampak sebagai plasenta yang
Dapat berkembang kista theca lutein (ovarian cyst), menebal dan multikistik disertai fetus atau jaringan
diameternya >6cm. Level -hCG sangat tinggi sehingga fetus. Pada kehamilan awal, tampakan ini sulit
corpus luteum yang normalnya memang dipertahankan ditemukan. Misdiagnosa yang sering terjadi adalah
oleh -hCG, malah membesar menjadi kista. Kista dapat dengan aborsi inkomplit atau missed abortion. Kadang
diraba di daerah abdomen bawah (bisa di kanan juga keliru dengan hamil kembar atau uterine
maupun kiri) dan memberikan penampakan asimetris leiyomyoma dengan kista yang terdegenerasi.
pada ibu hamil. Kista theca lutein tidak perlu diambil
dengan surgery. Jika Mola sudah diangkat, maka kista
theca lutein akan regresi dengan sendirinya tapi butuh
waktu 12 minggu. Kista theca lutein butuh diangkat bila
terjadi torsi yang dapat menyebabkan infark.
Gambar kista theca lutein di USG:
TERAPI normal (hingga periode follow up selesai, 6 bulan kadar -
Metode utamanya adalah operasi. hCG nya normal terus). Jika kadar -hCG sudah normal,
Jika pasien Mola masih menginginkan untuk memiliki anak, injeksi progesteron dapat digunakan. Penggunaan IUD tidak
metode suction curette adalah metode yang dipilih. Jika dianjurkan pada pasien hingga level -hCG tidak dapat
cervix pasien belum terbuka, maka dapat didilatasikan dideteksi.
dengan metode laminaria (memasukkan batang kecil yang
dapat menyerap air). Ada contoh kasus, seorang pasien post Molla datang ke
Jika pasien sudah tidak menginginkan untuk memiliki anak dokter umum di desa karena follow up yang gagal, perut
kembali, metode hysterectomy (pengangkatan rahim) membesar. Dari sana ia dirujuk ke dokter di kota, langsung
dapat dipilih karena mengurangi risiko rekurensi maupun dirujuk ke dokter spesialis di RS besar, kemudian diterapi
komplikasi. Hal ini terutama disarankan pada pasien mola dengan metrotrexat. Setelah dua kali mendapat terapi
comple yang sudah sangat besar. metrotrexat, barulah dilakukan USG dan ternyata
Sebelum dilakukan tindakan operasi (currete maupun ditemukan gestational sac berikut fetus di dalamnya.
hysterectomy), harus perhatikan ada tidaknya komplikasi Ternyata pasien tersebut hamil. Metrotrexat adalah anti
dari gejala tiroid seperti tyroid storm (apneu, kejang) asam folat yang dapat mencegah perkembangan
maupun insufiensi respirasi karena emboli tropoblast. tropoblast, sangat teratogenik. Sehingga bayi pasien
Pasien harus distabilkan sebelum dimulai tindakan. tersebut mengalami kecacatan, sehingga harus diterminasi.
Mola berkaitan dengan banyak sekali vaskularisasi, Nah, maka dari itu, besok kita harus hati-hati dalam
sehingga saat melakukan tindakan persediaan darah harus membuat diagnosis dan memberikan terapi. Semoga kita ga
diperhatikan dan pasien harus diberi infus. kayak gitu yah. Aamiin u.u

PERKEMBANGAN PENYAKIT Yak, sekian dulu materinya, semoga bermanfaat~~


Pasien post Mola berisiko untuk mengalami GTN
(Gestational Trophoblastic Neoplasia). GTN berkembang ECTOPIC PREGNANCY
dari 15-20% Mola komplit dan 5 % Mola parsial, dapat dr. Irwan Taufiqur Rachman, Sp.OG(K)
bermetastasis hingga paru. Bagi wanita yang mengalami
Pemateri: Indhah K | Editor: Annisa
GTN, nya mengalami Molar disease invasif lokal, dan
nya berkembang menjadi metastases (bisa ke hepar,
paru,dan otak). Perdarahan vagina adalah gejala paling ECTOPIC PREGNANCY
umum yang ditemukan pada diagnosis GTN pasca abortus Blastocyst normalnya berimplantasi di endometrial lining
atau terapi terminasi kehamilan (post Mola). Kriteria dari cavum uteri. Yang dimaksud ectopic pregnancy adalah
diagnosis untuk GTN dapat dilihat di tabel di bawah: implantasi di luar tempat tersebut. Sebanyak 95% dari
kehamilan ektopik terletak di sepanjang tuba, dan paling
banyak di ampula tuba (ampula 70%, fimbria 11%, isthmus
12%, cornu 2-3%). Sementara itu, 5% sisanya di ovarium,
cavitas peritoneum, di antara ligamentum latum, atau di
cervix. Jika diagnosis terlambat maka kehamilan ektopik
Follow Up terhadap pasien Mola sangat diperlukan untuk dapat membahayakan nyawa ibu.
mencegah terjadinya GTN. Umumnya, level -hCG akan
kembali ke normal (<4IU/L) pada 2-3 bulan pasca tindakan.
Jika level hCG kembali ke batas normal kurang dari 56 hari
pasca pengangkatan rahim, maka follow up terus dilakukan
selama 6 bulan sejak operasi pengangkatan rahim. Namun
jika level hCG tidak kunjung kembali ke batas normal pada
56 hari, maka follow up harus terus dilakukan hingga 6
bulan setelah level hCG kembali ke normal. Pengecekan
level hCG pada follow up dilakukan setiap 2 minggu
sehingga pasien selalu terkontrol.
Dapat dicurigai terjadi GTN jika pasca penanganan Mola,
pasien mengalami vaginal bleeding dan level -hCG tidak
turun setelah 2-3 bulan. Biasanya diagnosis GTN yang
menjadi maligna dan butuh kemoterapi hanya melalui
diagnosis klinis (anamnesis dan pemeriksaan fisik). Epidemiologi
Namun kita juga harus hati-hati, pasien bisa hamil pada Menurut American College of Obstetricians and
periode follow up, sehingga mengacaukan hasil deteksi - Gynecologists (2008), 2 % kehamilan trimester pertama di
hCG yang digunakan untuk memonitor GTN. Maka, US adalah kehamilan ektopik. Resiko kematian dari
pemeriksaan penunjanga, seperti USG harus dilakukan kehamilan ektopik cukup besar. FYI, 50% kehamilan di
untuk memastikan ada tidaknya GTN. cavum abdomen berakhir dengan kematian ibu karena
rawan menyebabkan perdarahan yang hebat, karena
Untuk mencegah terjadinya kehamilan, alat kontrasepsi plasenta bisa menempel di sembarang organ dan jika
barier harus digunakan hingga level -hCG mencapai dilepas akan menyebabkan pendarahan hebat. Selain itu,
keberhasilan kehamilan akan menurun setelah seorang ibu hypovolemic tadi dan juga adanya muscle rigidity/
mengalami kehamilan ektopik. Semakin lama, insidensi muscle guarding maka harus segera diberi pertolongan
kehamilah ektopik makin meningkat, kemungkinan pertama untuk menstabilkan kondisi pasien.
penyebanya adalah: Yang khas dari ectopic pregnancy adalah nyeri goyang
a. Lifestyle yang semakin jelek cervix dan adanya bulging dari fornix posterior akibat
b. Meningkatnya prevalensi sexually transmitted disease, darah yang terkumpul pada excavatio rectouterina
terutama karena Chlamydia trachomatis (cavum Douglas). Terlebih lagi, jika darah di cavum
c. Metode diagnosis dini kehamilan ektopik yang semakin peritonii sangat banyak maka akan dapat
canggih menyebabkan iritasi diafragma yang akan
d. Popularitas alat kontrasepsi yang dapat memicu menyebabkan manifestasi berupa nyeri pada bahu/
kehamilan ektopik leher.

Etiologi Diagnosis
Setiap gangguan dari perjalanan zygot menuju cavum uteri a. Quantitative Gonadotropin Levels (-hCG).
dapat menyebabkan kehamilan ektopik. Misalnya saja Normalnya, -hCG akan mulai terdeteksi 2-4
pemakaian obat-obatan muscle relaxant, hal ini akan minggu setelah ovulasi (>25 mIU/mL) dan
mengganggu kontraksi dari tuba uterine dan silia-silianya, kadarnya akan menjadi 2 kali lipat setiap 48-72
sehingga gerakan zygot sangat lambat dan akhirnya jam (atau dengan kata lain meningkat minimal
menempel di tuba. 66% setiap 48 jam) hingga mencapai kadar
maksimal 10.000 mIU/mL. Pada kehamilan
Faktor Resiko ektopik:
1. Kelainan anatomis tuba falopi, termasuk kelainan 1. peningkatan kadar -hCG tidak terlalu signifikan ,
kongenital karena paparan diethylstilbestrol (DES). yaitu <50% dalam 48 jam
2. Memiliki riwayat kehamilan ektopik 2. kadarnya cenderung tetap (plateau), perubahan
3. Memiliki riwayat penyakit menular seksual/ infeksi totalnya <15%, seperti terlihat pada grafik di
tuba lainnya bawah. (garis yang lebih atas adalah yang normal,
4. Pelvic inflammatory disease (dapat menyebabkan yang bawah yang ektopik).
kerusakan tuba) 3. -hCG <1.500 mIU/mL disertai nyeri dan vaginal
5. Endometriosis (adanya jaringan endometrium di luar bleeding
cavum uteri) 4. -hCG 2.000 mIU/mL dan tidak ada gestational sac
6. Assisted reproductive technology pada transvaginal ultrasound (TVUS)
7. Usia >40 tahun
8. Riwayat aborsi
9. Merokok (dapat berkontribusi pada kerusakan silia
sehingga mengurangi motilitas tuba)
10. Kegagalan metode kontrasepsi. Misalnya tubal
sterilization, intrauterine devices, high-dose estrogen
emergency contraception, and progestin-only minipills.
Kalau pada kontrasepsi IUD yang disertai progestin,
progestin dapat menyebabkan tubal pregnancy karena
progestin akan menurunkan motilitas dari tuba uterina.

Sign and Symptom


Kehamilan ektopik dapat dibedakan menjadi stable
b. Kadar hemoglobin dan hematocrit. Pemeriksaan
ectopic pregnancy (belum pecah) dan ruptured
CBC harus dilakukan, dan pada kehamilan ektopik
unstable ectopic pregnancy. Biasanya kalau usia gestasi
rupture yang diagnosisnya belum tegak dapat
sudah >8 minggu sudah rupture.
dilakukan pemeriksaan CBC secara berkala. Kalau
Classic triad: amenorrhea (biasanya terlambat
terjadi penurunan hb/ hct secara akut maka dapat
menstruasi 1-2 mingu), pendarahan abnormal vagina
menandakan kehamilan ektopik rupture. Kalau
(biasanya spotting bleeding), dan nyeri abdominal/
unrupture maka nilai hb dan hct masih normal.
pelvic. Namun, tak jarang gejala dari ectopic pregnancy
c. Kadar progesterone. Kehamilan intrauterine yang
ini tidak jelas. Hanya 50% pasien yang mengalami
normal memiliki nilai progesterone serum 25
ketiga gejala tersebut sekaligus. Nyeri abdomen/ pelvic
ng/mL. Nilai<5 ng/mL mengindikasikan kehamilan
baru akan terasa berat ketika terjadi rupture (nyeri
nonviable atau kehamilan ektopik. Namun, nilai
tajam/ stabbed/ seperti dirobek).
progesterone belum cukup untuk menegakkan
Pemeriksaan fisik yang didapatkan tergantung apakah
diagnosis kehamilan ektopik.
kehamilan ektopik itu masih stabil atau sudah rupture.
d. Transvaginal sonography (TVS).
Kalau sudah rupture dan perdarahan berlanjut, maka
Merupakan metode diagnosis yang baik untuk
akan muncul tanda shock hipovolemik, termasuk
ectopic pregnancy dan merupakan bagian dari
takikardi, hipotensi, dan penurunan kesadaran. Jika
algoritma diagnosis kehamilan ektopik. Hal ini
terjadi tanda kegawatdaruratan seperti tanda shock
digambarkan pada bagan di bawah.
Gestational sac merupakan struktur yang dapat
pertama kali dilihat dengan TVS. Gestational sac
merupakan lapisan echogenic/ putih (merupakan
reaksi desidual tropoblastik) yang mengelilingi
bagian tengah yang sonolucent/ hitam
(merupakan chorionic cavity). Dari tampakan TVS,
dapat diklasifikasikan (ini tambahan aja ya):
1. Definite intrauterine pregnancy: tampak
gestational sac dengan diameter>5mm di Gambar di atas menunjukkan teknik untuk
endometrium dan terdapat fetal pole dan/ mengidentifikasi hemoperitoneum(darah di cavitas
atau yolk sac di dalamnya. peritonii). Gambar A menunjukkan TVS. Terlihat
2. Probable abnormal intrauterine pregnancy: kumpulan cairan (anechoic atau sama dengan hitam)
gestational sac diameter>10 mm tanpa ada pada excavatio rectouterina. Gambar B menunjukkan
fetal pole, atau ada fetal pole tapi tidak culdocentesis atau prosedur untuk mengambil cairan
tampak aktivitas jantung. pada cavitas tersebut melalui fornix posterior
3. Presumed ectopic pregnancy: uterus kosong menggunakan spinal needle dan syringe. Jika memang
padahal kadar beta hCG melebihi cutoff value. terdapat darah di excavatio rectouterina maka
4. Definite ectopic pregnancy: terdapat struktur intervensi operatif diperlukan karena kemungkinan
bulat di luar uterus, dengan gestational sac kehamilan ektopik rupture tinggi.
yang mengandung fetal pole dan/ atau yolk f. Laparoskopi. Bisa menjadi pilihan diagnosis
sac di dalamnya terakhir karena invasive, namun dapat melihat
secara detail kehamilan ektopik. Laparoskopi
dapat sekaligus menjadi cara terapi.

Terapi
Terapi yang dapat dilakukan meliputi terapi medis (dengan
obat) dan terapi operatif. Terapi medis biasanya
menghasilkan angka mortilitas dan morbiditas yang lebih
rendah daripada terapi operatif, serta memiliki prognosis
fertilitas yang lebih baik daripada terapi operatif.
a. Medis: Methotrexate.
Dapat dilakukan jika:
1. Keadaan hemodinamik masih stabil.
2. Terdapat kenaikan -hCG setelah dilakukan kuret
dengan puncak <15.000 mIU/mL.
3. TVS menunjukkan unruptured ectopic pregnancy
dengan diameter <3,5 cm.
4. Masih menginginkan fertilitas.
5. Lapisan serosa dari tuba masih intak.
---
Methotrexate (MTX)
Mekanisme aksi: sebagai antagonis asam folat,
menginaktivasi dihydrofolate reductase menyebabkan
kekurangan tetrahydrofolate, yang dibutuhkan untuk
sintesis DNA dan RNA. Hasilnya adalah menghambat
pertumbuhan sel tropoblast pada ectopic pregnancy.
---
Penjelasan grafik: jika ada pasien dengan tes kehamilan
Kontraindikasi terapi MTX:
positif datang dan mengeluh nyeri dan perdarahan,
1. Ibu menyusui
maka yang pertama kali dilakukan adalah menilai
2. Alkoholisme, alcoholic liver disease, atau penyakit
apakah keadaan hemodinamiknya stabil atau tidak. Jika
liver kronis lain
tidak stabilkan kondisi dan jika di rumah sakit besar
3. Disfungsi hepar: aspartate aminotransferase>2 kali
bisa langsung dilakukan operasi. Jika stabil lakukan
nilai normal
TVS, maka akan terlihat kehamilan ektopik/ kehamilan
4. Penyakit ginjal: kadar kreatinin serum >130
normal/ abortus lakukan manajemen yang sesuai.
mmol/L (1.5mg/dL). Oleh karena itu, pada
Jika diagnosis belum bisa dibuat (tidak jelas) maka
pemeriksaan kehamilan ektopik juga perlu dinilai
lakukan pemeriksaan serum beta-hCG, dan seterusnya
kadar kreatinin dan liver transaminase untuk
(ini nggak terlalu dijelasin sampe akhir guys).
mengetahui apakah ada kontraindikasi.
e. Culdocentesis 5. Active peptic ulcer disease
6. Blood dyscrasia: leukocyte count <3,000 cells/uL
atau platelet count 100,000/uL
7. Embryonic cardiac motion
Monitoring Terapi - Abortus Provocatus
1. Konsentrasi -hCG sering meningkat setelah injeksi Berbeda dengan abortus yang spontan (dibahas di kuliah
pertama MTX lain), abortus provokatus (indoset abortion) adalah aborsi
2. Kadar -hCG normalnya turun 15% dari hari yang disengaja baik dengan memakai obat-obatan maupun
keempat sampai hari ketujuh setelah administrasi alat-alat, ini terbagi menjadi dua:
MTX. a. Abortus provocatus medicinalis, adalah aborsi yang
3. TVUS bukan metode yang tepat untuk dilakukan oleh dokter atas dasar indikasi medis, yaitu
menentukan kegagalan terapi. apabila tindakan aborsi tidak diambil akan
4. Terkadang setelah terapi sering ditemukan membahayakan jiwa ibu
pembesaran massa ektopik dan menyebabkan b. Abortus provocatus criminalis, adalah aborsi yang
tindakan yang tidak perlu dilakukan. terjadi oleh karena tindakan-tindakan yang tidak legal
b. Operasi atau tidak berdasarkan indikasi medis, sebagai contoh
Dilakukan jika: aborsi yang dilakukan dalam rangka melenyapkan janin
1. Keadaan hemodinamik tidak stabil atau terjadi sebagai akibat hubungan seksual di luar perkawinan.
kegagalan terapi medikasi (treatment failure).
Treatment failure didefinisikan sebagai MEDIS VS HUKUM
kebutuhan untuk dilakukan intervensi operatif. Dari segi pengertian mengenai aborsi, medis dan hukum
Ada juga yang merujuk kepada kegagalan 1 kali mempunyai perbedaan. Dari segi hukum, mereka tidak
injeksi MTX untuk menurunkan angka -hCG berpikir ke depan mengenai janin dapat hidup di luar
sebanyak 15%. kandungan. Maksudnya apa? Jadi kalo dari segi hukum
2. -hCG > 15.000 mIU/mL aborsi itu ya menggugurkan kandungan dengan sengaja
3. TVS menunjukkan kehamilan ektopik rupture entah si bayi nantinya bisa hidup di luar kandungan
4. Enough children ataupun nggak setelah ia seharusnya dilahirkan.
5. Lapisan serosa dari tuba tidak intak. Abortus dari sisi hukum kita akan belajar mengenai
6. Juga diindikasikan untuk pasien yang memiliki perbedaan abortus yang legal dan tidak legal.
riwayat kehamilan ektopik pada tuba yang sama. Berapa usia kehamilan yang diijinkan untuk dilakukan
Operasi yang dimaksud adalah salpingostomy, aborsi?
salpingectomy, partial salpingectomy, segmental
resection, atau cornual resection. UU Kesehatan (36/2009) tentang kesehatan
Penjelasan: (tambahan) Pasal 75
salpingostomy Melubangi tuba uterine untuk (1) Setiap orang dilarang melakukan aborsi.
mengeluarkan massa ektopik. (2) Larangan sebagaimana dimaksud pada ayat (1)
Dilakukan jika diameter massa dapat dikecualikan berdasarkan:
ektopik<2 cm dan terletak di 1/3 a. Indikasi kedaruratan medis yang dideteksi sejak
distal tuba uterine usia dini kehamilan, baik yang mengancam
salpingectomy Mengangkat tuba uterine. Bisa nyawa ibu dan/atau janin, yang menderita
untuk yang ruptured/ penyakit genetik berat dan/atau cacat bawaan,
unrupture. maupun yang tidak dapat diperbaiki sehingga
menyulitkan bayi tersebut hidup di luar
kandungan; atau
LEGAL ASPECT OF CRIMINAL INDUCED b. Kehamilan akibat perkosaan yang dapat
menyebabkan trauma psikologis bagi korban
ABORTION perkosaan
dr. Beta Ahlam Gizela, Sp.F, DFM Note: untuk indikasi medis yang disebutkan pada
ayat ini, saat ini penegakan diagnosis berupa cacat
Pemateri: Retty | Editor: Rara kongenital sudah dapat diliat dari dalam kandungan
(minggu awal kehamilan). Indikasi medis yang
Berdoa dulu ya sebelum belajar
disebut diatas adalah indikasi medis yang berakibat
ABORSI fatal untuk ibu dan atau bayi itu sendiri. Sebagai
Materi yang akan disampaikan kali ini memang sebagian besar contoh : apabila ditemukan cacat kongenital dapat
berisi tentang aturan dan undang-undang mengenai aborsi. dilakukan terminasi, contoh pada kasus anencephaly
Sebenarnya apa sih aborsi itu ? (harus ijin ibu + suaminya dulu ya mau nggak
Aborsi adalah tindakan menggugurkan kandungan diterminasi). Nah, untuk kasus aborsi kehamilan
pengeluaran hasil konsepsi (pertemuan sel telur dan sel akibat perkosaan ini kasusnya banyak. TAPI, anehnya
sperma) sebelum janin dapat hidup di luar kandungan. permintaan aborsi akibat perkosaan ini malah
Dengan kata lain, aborsi adalah suatu proses pengakhiran diminta ketika kehamilan sudah memasuki bulan
hidup dari janin sebelum diberi kesempatan untuk lanjut >5 bulan. Pertanyaannya: kenapa nggak dari
bertumbuh. Bedanya dengan abortus itu apa sih? Kalau dulu aja diaborsi kalo emang itu BENERAN karena
aborsi dilakukan dengan sengaja, kalau abortus tidak perkosaan? Nah dari sini emang masih banyak
disengaja.
aturan lagi ya untuk dipatuhi dan diselidiki (terutama PP 61/2014 Tentang Kesehatan Reproduksi
untuk psikologisnya). o Pasal 31
(3) Tindakan sebagaimana dimaksud pada ayat (2) (1) Tindakan aborsi hanya dapat dilakukan berdasarkan:
hanya dapat dilakukan setelah melalui konseling a. indikasi kedaruratan medis; atau b. kehamilan
dan/atau penasehatan pra tindakan dan diakhiri akibat perkosaan
dengan konseling pasca tindakan yang dilakukan (2) Tindakan aborsi akibat perkosaan sebagaimana
oleh konselor yang kompeten dan berwenang. dimaksud pada ayat (1) huruf b hanya dapat
Note: konseling dilakukan oleh konselor yang dilakukan apabila usia kehamilan paling lama
KOMPETEN yaa, jadi nggak semua dokter bisa jadi berusia 40 (empat puluh) hari dihitung sejak hari
konselor. pertama haid terakhir
(4) Ketentuan lebih lanjut mengenai indikasi Note: sedikit berbeda dengan undang-undang yang
kedaruratan medis dan perkosaan, sebagaimana tadi, pada PP 61/2014 pasal 31 ayat (2) ini usia
dimaksud pada ayat (2) dan ayat (3) diatur dengan kehamilan yang diperbolehkan untuk diaborsi adalah
Peraturan Pemerintah. 40 hari, bukan 6 minggu (42 hari).
Pasal 76 o Pasal 32
Aborsi sebagaimana dimaksud dalam Pasal 75 hanya (1) Indikasi kedaruratan medis sebagaimana dimaksud
dapat dilakukan: dalam Pasal 31 ayat (1) huruf a meliputi: a.
a. sebelum kehamilan berumur 6 (enam) minggu kehamilan yang mengancam nyawa dan kesehatan
dihitung dari hari pertama haid terakhir, kecuali ibu; dan/atau b. kehamilan yang mengancam nyawa
dalam hal kedaruratan medis; dan kesehatan janin, termasuk yang menderita
note: pada kasus perkosaan, abortus dilakukan pada penyakit genetik berat dan/atau cacat bawaan,
kehamilan yang berumur <6 minggu. Akan tetapi, maupun yang tidak dapat diperbaiki sehingga
berbeda dengan kasus indikasi medis, aborsi dapat menyulitkan bayi tersebut hidup di luar kandungan
dilakukan setelah penegakan diagnosis secara pasti (2) Penanganan indikasi kedaruratan medis
karena untuk kasus kelainan kongenital biasanya sebagaimana dimaksud pada ayat (1) dilaksanakan
baru bisa dilihat secara jelas setelah kehamilan sesuai dengan standar.
berumur 6 minggu. o Pasal 33
b. oleh tenaga kesehatan yang memiliki keterampilan (1) Penentuan adanya indikasi kedaruratan medis
dan kewenangan serta memiliki sertifikat yang sebagaimana dimaksud dalam Pasal 32 dilakukan
ditetapkan oleh menteri kesehatan; oleh tim kelayakan aborsi
note: pada bagian ini dijelaskan siapa sih yang boleh Note: ada tim yang menilai kelayakan tindakan
melakukan tindakan aborsi. Dapat digarisbawahi aborsi. Tim ini bersifat terpisah dan bukan terdiri
kata keterampilan dan kewenangan yaa. Tenaga dari dokter yang melakukan tindakan aborsi.
kesehatan yang boleh melakukan adalah yang (2) Tim sebagaimana dimaksud pada ayat (1) paling
KOMPETEN dan BERWENANG. Kalo cuma kompeten sedikit terdiri dari 2 (dua) orang tenaga kesehatan
aja tapi nggak berwenang aka punya ijin/tugas ya yang diketuai oleh dokter yang memiliki kompetensi
nggak boleh. Contoh: semua dokter obsgyn bisa dan kewenangan
melakukan aborsi TAPI nggak semua dokter obsgyn Note: ketua tim kelayakan aborsi harus memiliki
boleh melakukannya. Namun, ada aturan berbeda kewenangan ya, nggak sembarang dokter di rumah
ketika memang nggak ada dokter lain (disesuaikan sakit bisa jadi ketua tim kelayakan aborsi (ada orang
dengan aturan yang ada). khusus yang diberi tugas)
c. dengan persetujuan ibu hamil yang bersangkutan; (3) Dalam menentukan indikasi kedaruratan medis, tim
d. dengan izin suami, kecuali korban perkosaan; dan sebagaimana dimaksud pada ayat (1) harus
e. penyedia layanan kesehatan yang memenuhi syarat melakukan pemeriksaan sesuai dengan standar
yang ditetapkan oleh menteri. Note: jadi, kalo untuk menentukan adanya bahaya
Pasal 77 pada ibu/janin dan adanya kemungkinan bahwa
Pemerintah wajib melindungi dan mencegah janin tidak bisa hidup di luar kandungan maka harus
perempuan dari aborsi yang tidak bermutu, tidak aman, dilakukan pemeriksaan dengan GOLD STANDARD
dan tidak bertanggung jawab serta bertentangan yang sudah ditentukan.
dengan norma agama dan ketentuan peraturan (4) Berdasarkan hasil pemeriksaan sebagaimana
perundang-undangan sebagaimana dimaksud dalam dimaksud pada ayat (3), tim sebagaimana dimaksud
Pasal 75 ayat (2) dan ayat (3). pada ayat (1) membuat surat keterangan kelayakan
Note: perlindungan yang diberikan (termasuk dengan aborsi.
cara dikeluarkannya aturan/undang-undang ini) o Pasal 34
bertujuan untuk mencegah adanya pelanggaran hukum, (1) Kehamilan akibat perkosaan sebagaimana dimaksud
etik dan agama. Selain itu, dengan adanya aturan yang dalam Pasal 31 ayat (1) huruf b merupakan
jelas diharapkan masyarakat (pasien tentunya) dapat kehamilan hasil hubungan seksual tanpa adanya
terjamin keselamatannya. persetujuan dari pihak perempuan sesuai dengan
ketentuan peraturan perundang-undangan
(2) Kehamilan akibat perkosaan sebagaimana dimaksud
pada ayat (1) dibuktikan dengan: a. usia kehamilan
sesuai dengan kejadian perkosaan, yang dinyatakan (2) Dokter sebagaimana dimaksud pada ayat (1) bukan
oleh surat keterangan dokter; dan b. keterangan merupakan anggota tim kelayakan aborsi atau
penyidik, psikolog, dan/atau ahli lain mengenai dokter yang memberikan surat keterangan usia
adanya dugaan perkosaan kehamilan akibat perkosaan
Note: harus bisa dibuktikan kalau kehamilan Note: kenapa dokternya harus beda? Karena biar
tersebut dikarenakan hasil perkosaan atau hasil nggak ada conflict of interest. Bisa aja kan dokter
hubungan seksual tanpa adanya persetujuan dari memberikan keterangan usia kehamilannya dibuat-
pihak perempuan. Bukti yang diperlukan tentu aja buat biar bisa tetep melakukan tindakan aborsi.
banyak ya. Pembuktian tindakan perkosaan itu (3) Dalam hal di daerah tertentu jumlah dokter tidak
terkadang sulit karena kan biasanya perkosaan itu mencukupi, dokter sebagaimana dimaksud pada
tidak ada saksinya, kecuali emang ada CCTV atau ayat (1) dapat berasal dari anggota tim kelayakan
perkosaannya itu nggak dilakukan oleh 1 orang saja aborsi
dalam waktu bersamaan (berkelompok gitu ngeri Note: ini kalo di pedalaman/di tempat antah
._.). Keterangan dari penyidik aka polisi dibutuhkan berantah yang mana dokternya emang terbatas yaa.
untuk menggali keterangan yang ada, selain itu (4) Ketentuan lebih lanjut mengenai pelatihan
untuk membuktikan bahwa keterangan yang sebagaimana dimaksud pada ayat (1) diatur dengan
disampaikan oleh korban perkosaan tersebut Peraturan Menteri.
bohong atau tidak maka keahlian psikolog Note: ada daftarnya lho guys dokter yang bisa
diperlukan juga. Ahli lain yang dapat berperan melakukan tindakan aborsi itu siapa aja jadi besok kalau
adalah psikiater. DAN jangan lupa, usia kehamilan udah jadi dokter jangan ngaku-ngaku ya kalau sebenarnya
sesuai dengan waktu terjadinya perkosaan. Kenapa? gapunya izin hoho.
Soalnya beberapa kasus terjadi bahwa kehamilan o Pasal 37
sudah terjadi SEBELUM perkosaan itu terjadi (nah (1) Tindakan aborsi berdasarkan indikasi kedaruratan
lho?) medis dan kehamilan akibat perkosaan hanya dapat
o Pasal 35 dilakukan setelah melalui konseling
(1) Aborsi berdasarkan indikasi kedaruratan medis dan (2) Konseling sebagaimana dimaksud pada ayat (1)
kehamilan akibat perkosaan harus dilakukan dengan meliputi konseling pra tindakan dan diakhiri dengan
aman, bermutu, dan bertanggung jawab konseling pasca tindakan yang dilakukan oleh
(2) Praktik aborsi yang aman, bermutu dan bertanggung konselor.
jawab sebagaimana dimaksud pada ayat (1) Note: Adapun tujuan pra konseling adalah (Pasal 37
meliputi: a. dilakukan oleh dokter sesuai dengan ayat (3) PP 61/2014):
standar; b. dilakukan di fasilitas pelayanan a. menjajaki kebutuhan dari perempuan yang ingin
kesehatan yang memenuhi syarat yang ditetapkan melakukan aborsi;
oleh Menteri; c. atas permintaan atau persetujuan b. menyampaikan dan menjelaskan kepada
perempuan hamil yang bersangkutan; d. dengan izin perempuan yang ingin melakukan aborsi bahwa
suami, kecuali korban perkosaan; e. tidak tindakan aborsi dapat atau tidak dapat dilakukan
diskriminatif; dan f. tidak mengutamakan imbalan berdasarkan hasil pemeriksaan klinis dan
materi pemeriksaan penunjang;
Note: yang penting disini adalah tindakan aborsi ini c. menjelaskan tahapan tindakan aborsi yang akan
dilakukan di fasilitas kesehatan yang memenuhi dilakukan dan kemungkinan efek samping atau
syarat yang ditetapkan oleh menteri. BUKAN di komplikasinya;
praktik pribadi yaa, tapi di rumah sakit. Kalo di d. membantu perempuan yang ingin melakukan
praktik pribadi sudah bisa disebut sebagai tindakan aborsi untuk mengambil keputusan sendiri untuk
aborsi yang ilegal. melakukan aborsi atau membatalkan keinginan
(3) Dalam hal ini perempuan hamil sebagaimana untuk melakukan aborsi setelah mendapatkan
dimaksud pada ayat (2) huruf c tidak dapat informasi mengenai aborsi; dan
memberikan persetujuan, persetujuan aborsi dapat e. menilai kesiapan pasien untuk menjalani aborsi.
diberikan oleh keluarga yang bersangkutan Sedangkan konseling pasca tindakan dilakukan
Note: contoh kasusnya pada kondisi darurat dimana dengan tujuan (Pasal 37 ayat (4) PP 61/2014):
ibu sudah tidak sadar sepenuhnya ataupun tidak a. mengobservasi dan mengevaluasi kondisi pasien
kompeten dalam memberikan keputusan, inform setelah tindakan aborsi;
consent dapat diberikan kepada keluarganya. b. membantu pasien memahami keadaan atau
(4) Dalam hal suami tidak dapat dihubungi, izin kondisi fisik setelah menjalani aborsi;
sebagaimana dimaksud pada ayat (2) huruf d c. menjelaskan perlunya kunjungan ulang untuk
diberikan oleh keluarga yang bersangkutan. pemeriksaan dan konseling lanjutan atau
o Pasal 36 tindakan rujukan bila diperlukan; dan
(1) Dokter yang melakukan aborsi berdasarkan indikasi d. menjelaskan pentingnya penggunaan alat
kedaruratan medis dan kehamilan akibat perkosaan kontrasepsi untuk mencegah terjadinya
sebagaimana dimaksud dalam Pasal 35 ayat (2) kehamilan.
huruf a harus mendapatkan pelatihan oleh Note: konseling penting banget lho untuk dilakukan
penyelenggara pelatihan yang terakreditasi karena bisa aja setelah pasien tau risiko yang bisa
terjadi karena tindakan aborsi, pasien tidak jadi sebenarnya. Namun, nilai ini dikonversi dengan
melakukan tindakan aborsi. harga emas sekarang.
o Pasal 38 2. Jika yang bersalah, berbuat demikian untuk mencari
(1) Dalam hal korban perkosaan memutuskan keuntungan, atau menjadikan perbuatan tersebut
membatalkan keinginan untuk melakukan aborsi sebagai pencarian atau kebiasaan, atau jika dia
setelah mendapatkan informasi mengenai aborsi seorang tabib, bidan atau juru obat, pidananya
sebagaimana dimaksud dalam Pasal 37 ayat (3) dapat ditambah sepertiga
huruf d atau tidak memenuhi ketentuan untuk 3. Jika yang bersalah, melakukan kejahatan tersebut,
dilakukan tindakan aborsi sebagaimana dimaksud dalam menjalani pencarian maka dapat dicabut
dalam Pasal 31 ayat (2), korban perkosaan dapat haknya untuk melakukan pencarian itu.
diberikan pendampingan oleh konselor selama masa Note: pencarian disini maksudnya mata pencaharian
kehamilan aka STR aka surat ijin praktik kalo buat dokter.
(2) Anak yang dilahirkan dari ibu korban perkosaan o Pasal 346
sebagaimana dimaksud pada ayat (1) dapat diasuh Seorang wanita yang sengaja menggugurkan atau
oleh keluarga mematikan kandungannya atau menyuruh orang lain
(3) Dalam hal keluarga sebagaimana dimaksud pada untuk itu, diancam dengan pidana penjara paling lama
ayat (2) menolak untuk mengasuh anak yang empat tahun.
dilahirkan dari korban perkosaan, anak menjadi o Pasal 347
anak asuh yang pelaksanaannya dilakukan sesuai (1) Barangsiapa dengan sengaja menggugurkan atau
dengan ketentuan peraturan perundang-undangan mematikan kandungan seorang wanita tanpa
Note: peraturan mengenai pengasuhan anak ini persetujuannya, diancam dengan pidana penjara
dimaksudkan untuk perlindungan dan kejelasan paling lama dua belas tahun
identitas si anak. Kalo nggak ada aturan atau (2) Jika perbuatan itu mengakibatkan matinya wanita
kejelasan hukum bisa jadi kasus penggelapan tersebut, dikenakan pidana penjara paling lama lima
identitas. Contoh kasus: untuk menghindari adanya belas tahun
kemungkinan perkawinan satu darah atau Note: kalo ibunya meninggal sudah pasti lebih berat
ketidakjelasan hak waris. ya karena kan dianggap melakukan pembunuhan
o Pasal 39 terhadap 2 orang (ibu dan anak).
(1) Setiap pelaksanaan aborsi wajib dilaporkan kepada o Pasal 348
kepala dinas kesehatan kabupaten/kota dengan (1) Barangsiapa dengan sengaja menggugurkan atau
tembusan kepala dinas kesehatan provinsi mematikan kandungan seorang wanita dengan
(2) Laporan sebagaimana dimaksud pada ayat (1) persetujuannya, diancam dengan pidana penjara
dilakukan oleh pimpinan fasilitas pelayanan paling lama lima tahun enam bulan
kesehatan (2) Jika perbuatan itu mengakibatkan matinya wanita
Note: dari sini kita bisa tahu yaa seharusnya tiap tersebut, dikenakan pidana penjara paling lama
tahun ada laporan aka angka resmi dilakukannya tujuh tahun.
tindakan aborsi. Alur singkatnya: laporan dari tim o Pasal 349
kelayakan aborsi pimpinan rumah sakit aka Jika seorang tabib, bidan atau juru obat membantu
direksi dinas kesehatan kota/kabupaten dinas melakukan kejahatan yang tersebut pasal 346, ataupun
kesehatan provinsi. melakukan atau membantu melakukan salah satu
kejahatan yang diterangkan dalam pasal 347 dan 348,
HUKUM ABORSI maka pidana yang ditentukan dalam pasal itu dapat
- Menurut hukum yang berlaku di Indonesia, aborsi atau ditambah dengan sepertiga dan dapat dicabut hak
pengguguran janin termasuk kejahatan, yang dikenal untuk menjalankan pencarian dalam mana kejahatan
dengan istilah Abortus Provocatus Criminalis. Yang dilakukan.
menerima hukuman adalah:
Ibu yang melakukan aborsi RESIKO KESEHATAN DAN KESELAMATAN FISIK
Dokter atau bidan atau dukun yang membantu Pada saat melakukan aborsi dan setelah melakukan aborsi
melakukan aborsi ada beberapa resiko yang akan dihadapi seorang wanita,
Orang-orang yang mendukung terlaksananya aborsi. seperti yang dijelaskan dalam buku Facts of Life yang
ditulis oleh Brian Clowes, Phd yaitu:
Hukum Aborsi dalam KUHP Kematian mendadak karena pendarahan hebat
o Pasal 229 Kematian mendadak karena pembiusan yang gagal
1. Barang siapa dengan sengaja mengobati seorang Kematian secara lambat akibat infeksi serius
wanita atau menyuruhnya supaya diobati, dengan disekitar kandungan
diberitahukan atau ditimbulkan harapan, bahwa Rahim yang sobek (uterine perforation)
karena pengobatan itu hamilnya dapat digugurkan, Kerusakan leher rahim (cervical lacerations) yang
diancam dengan pidana penjara paling lama empat akan menyebabkan cacat pada anak berikutnya
tahun atau denda paling banyak tiga ribu rupiah Kanker payudara (karena ketidakseimbangan
Note: karena KUHP ini dibuat pada tahun 1981 hormon estrogen pada wanita)
kayanya dokternya typo deh, 1918 kan belum merdeka masa ada
Kanker indung telur (ovarian cancer)
KUHP hehe, maka tiga ribu rupiah disini bukanlah nilai
Kanker leher rahim (cervical cancer) Sedangkan garis kedua mengandung antibody anti beta
Kanker hati (liver cancer) hCG, yang merupakan subunit hormon hCG yang lebih
Kelainan pada placenta/ari-ari (placenta previa) yang spesifik hCG. Sekarang ada strip test yang menaruh anti
akan menyebabkan cacat pada anak berikutnya dan alpha hCG (kontrol) nya pada garis horizontal dan anti beta
pendarahan hebat pada saat kehamilan berikutnya hCG di garis vertical, sehingga ketika tidak ada hormon hCG
Menjadi mandul/tidak mampu memiliki keturunan dalam urin akan terbentuk garis horizontal (-), sedangkan
lagi (ectopic pregnancy) kalau mengandung hormon hCG dalam urin akan
Infeksi rongga panggul (Pelvic Inflammatory Disease) membentuk garis horizontal dan vertical (+). Strip test jenis
Infeksi pada lapisan rahim (endometriosis). ini lebih mahal, bisa mencapai 90000 rupiah. Sedangkan tes
aglutinasi menggunakan prinsip immunoassay juga, hanya
RISIKO KESEHATAN MENTAL saja, antibody dibuat dalam bentuk terdispersi dalam
Proses aborsi bukan saja suatu proses yang memiliki resiko medium cair, sehingga butuh lebih banyak antibodinya.
tinggi dari segi kesehatan dan keselamatan seorang wanita Nah complex antibody-antigen akan membentuk aglutinasi
secara fisik, tetapi juga memiliki dampak yang sangat hebat berupa granul. Nah harganya lebih mahal, sekitar 300000
terhadap keadaan mental seorang wanita. rupiah. Sejarah tes kehamilan sudah berkembang sejak
Gejala ini dikenal dalam dunia psikologi sebagai Post- lama.
Abortion Syndrome (Sindrom Paska-Aborsi) atau PAS. - 1350 BC, jaman Mesir Kuno mengasumsi hamil atau
Gejala-gejala ini dicatat dalam Psychological Reactions tidak dengan menyuruh wanita pipis di biji barley dan
Reported After Abortion di dalam penerbitan The Post- gandum. Nah kalau yang tumbuh barley, berarti
Abortion Review (1994). bayinya cowok. Kalo yang tumbuh gandum, berarti
Pada dasarnya seorang wanita yang melakukan aborsi akan bayinya cewek; kalo ga ada yang tumbuh berarti ga
mengalami hal-hal seperti berikut ini: hamil. Nah kalo keduanya tumbuh? Saya juga tidak
Kehilangan harga diri (82%) tahu..; kalo ga ada yang tumbuh, berarti ga hamil.
Berteriak-teriak histeris (51%) Uniknya pada tahun 1963 hal ini diteliti lagi dan
Mimpi buruk berkali-kali mengenai bayi (63%) 70% percobaan berhasil. Kemungkinan dalam urin
Ingin melakukan bunuh diri (28%) wanita hamil mengandung sejenis growth factor
yang mampu menumbuhkan tanaman tadi.
Mulai mencoba menggunakan obat-obat terlarang
- Pada middle age, sekitar tahun 1552, Kalo urin wanita
(41%)
hamil ditambah ke wine bakal kayak ada warna awan
Tidak bisa menikmati lagi hubungan seksual (59%)
putih sari lemon gitu. Hal ini dapat disebabkan karena
Diluar hal-hal tersebut diatas para wanita
wine mengandung alcohol dan urine wanita hamil
yangmelakukan aborsi akan dipenuhi perasaan
mengandung glikoprotein yang banyak yang
bersalah yang tidak hilang selama bertahun-tahun
menyebabkan terbentuknya awan berwarna putih.
dalam hidupnya.
- Pada 1927, Ascheim dan Zondak melakukan teknik
penentuan kehamilan dengan bioassay pertama kali,
Guys, tindakan aborsi ini memang banyak aturan, baik buruknya,
dan masih banyak hal hal lainnya. Diharapkan ketika kalian dengan menyuntikkan urin wanita hamil trimester
nantinya berhadapan dengan kasus seperti ini dapat pertama ke tikus yang immature, tikus tersebut akan
mempertimbangkan segala sesuatunya termasuk menyelamatkan berubah menjadi matur (ovarium membesar dan sudah
ibu atau bayi? Bagaimana norma etik, agama, dan sosial atau ada folikel matur). Tingkat kesalahannya <2%.
bahkan ekonominya? Bagaimana psikologisnya? Dan masih Tes ini dikembangkan oleh Friedman pada tahun
banyak lainnya. Maaf tambahannya nggak banyak soalnya 1931, dengan teknik serupa tapi pada kelinci. Pada
bingung referensinya dari mana dan mau berdiskusi ataupun rabbit test ini, sudah dikemukakan teori bahwa
berdebat sama siapa. Hehehe
ada hormon yang menginduksi pematangan ini
yang dikeluarkan ibu hamil pada trimester 1, yang
PRAKTIKUM FISIOLOGI: DETECTION OF HCG IN nantinya ditemukan sebagai hCG.
URINE Tes ini dikembangkan lagi pleh Galli Mainiini pada
Pemateri: Ervin | Editor: Amani tahun 1952 yang menggunakan katak Batrachias
jantan, maka spermatozoa akan terbentuk, dan
pada katak betina akan terjadi maturasi sehingga
PENDAHULUAN akan bertelur. Cara ini mengurangi pengorbanan
Deteksi HCG yang kita lakukan sekarang di praktikum kelinci demi mengetes kehamilan.
fisiologi kali ini adalah sebuah immunoassay, yakni sebuah - Pada 1960, immunoassay untuk mengetes kehamilan
reaksi bertemunya antigen (dalam hal imi hCG) dengan pertama kali diperkenalkan dengan menggunakan
antibodinya. Hasilnya dapat berupa aglutinasi atau warna Hemagglutination inhibition test, yang menginjeksikan
karena antibody sudah diikat dengan zat warna. sel darah merah dan beta HCG. Cara ini murah, cepat,
Pada strip test, garis yang atas menunjukkan kontrol, tapi kurang sensitive.
karena mengandung antibody anti alfa hCG yang - 1970-1972, ditemukan bahwa hCG punya 2 subunit
merupakan subunit tidak spesifik hCG, yang mirip dengan alpha dan beta, dan yang beta punya efek seperti LH.
banyak subunit dari banyak hormon lain, seperti alpha TSH, Hal ini menjadi dasar pembentukan strip test. Sehingga
alpha LH, dan alpha FSH; sehingga nilainya akan selalu pada 1976 FDA memberi izin beredarnya tes
positif pada urin wanita tidak hamil maupun urin pria. kehamilan.
Selanjutnya ke fisiologi beta hCG. bhCG ini penting di Day 5-6 setelah fertilisasi, terjadi proses aposisi, yakni
trimester 1 atau awal kehamilan, karena ia penempelan embryo secara superfisial pada endometrium,
mempertahankan korpus luteum, sehingga progesterone sudah ada tropobals namun sedikit. Kadar hCG masih
tetap tinggi dan endometrium tetap terjaga, sebelum rendah (sekitar 5 mIU/ml), kadar ini merupakan kadar hCG
plasenta terbentuk. bhCG sendiri adalah glikoprotein yang hamil biokimia awal. Saat ini penempelan tidak lebih dari 5-
memliki rantai as. Amino yang lebih banyak daripada ahCG 6 mm, belum kuat, rapuh, rawan aborsi.
(145 > 92). Day 7-8 setelah fertilisasi, sudah terjadi proses adhesi,
Perlu diketahui bahwa hCG juga disekresikan secara yakni fase awal implantasi. hCG sudah dihasilkan sekitar 25-
tinggi ketika ada tumor malignan, sehingga ketika urin laki- 50 mIU / ml, dan sudah bisa terdeteksi strip test. Dengan
laki positif hCG maka curiga adanya seminoma testis, kalau kata lain kalo ada pasangan udah ga sabaran pingin punya
pada wanita non hamil harus curiga choricarcinoma. Pada anak, bisa melakukan strip test sebelum telat haid (5-6 hari
pria dan wanita tidak hamil sebenarnya hCG tetap sebelum tanggal haid), dan hasilnya bisa positif. Hal ini
disekresikan dalam jumlah kecil oleh pituitary. dikarenakan sincytiotropoblas sudah terbentuk, sehingga
Apa fungsi hCG saat kehamilan? hCG sudah lebih banyak tersekresi.
- Utama: mempertahankan corpus luteum sehingga bisa Day 8-9 sudah terjadi implantasi. Setelah itu
menghasilkan progesterone untuk mempertahankan syncitiotropoblas terus menghasilkan hCG sehingga
endometrium akhirnya endometrium tetap terjaga dan tidak terjadi
- Parakrin ke cytotropoblas, mempertahankannya dan menstruasi (~telat). Kadar hCG terus naik sehingga pada 3
menginduksi pembentukannya dan perkembangannya hari setelah telat kadarnya mencapai 200 mIU / ml yang
- Mengikis endometrium (sisa 1/3 saja) dan digantikan dapat diukur melalui tes aglutinasi. Tes ini lebih baik untuk
dengan desidua (bagian dari plasenta). Hal ini melihat implantasi sudah terjadi atau tidak, sehingga ketika
menyebabkan endometrium tidak penting lagi. Apa strip positif dan agglutinasi neatif, kemungkinan terjadi
maksudnya tidak penting? Artinya ketika bayi lahir, kegagalan implantasi.
semua bagian bayi harus lahir termasuk plasenta (dan Seperti yang telah kita ketahui, plasenta terbrntuk dari
desiduanya), dan endometrium tidak ikut lahir. Nah villi khorion. Day 13 terbentuk primary villi yang belum
dengan kata lain sebenarnya ketika plasenta sudah terinvaginasi, hari ke 16: seconfary villi, dan hari ke 21:
terbentuk, bayi dapat hidup dengan sedikit sekali udah terjadi difusi nutrisi dan zat buang. Saat melahirkan,
peran dari endometrium, yakni hanya menyediakan plasenta tidak boleh tertinggal, karena akan menyebabkan
nutrisi dan menyalurkan zat sisa. retained placenta, yakni kegagalan kontraksi uterus karena
- Menstimulasi testosterone fetus laki-laki (karena mirip ada plasenta di dalamnya, sehingga penrdarahan tidak
LH). Setelah 110 hari, bayi sudah bisa menghasilkan LH berhasil dihentikan perdarahan post partum. Hal ini juga
sendiri, sehingga ketika hCG turun pematangan meningkatkan resiko puerperal infection / postpartum
genitalia fetus laki-laki dapat berjalan. Pada fetus infection.
wanita dapat menginduksi pematangan folikular.
- Stimulasi tiroid maternal weeks since LMP mIU/mL
- Membantu vasodilatasi vascular (hemodilusi) 3 5 50
- Stimulasi pengeluaran relaxin. 4 5 426
Nah sensitivitas strip test itu sendiri adalah 25 mIU/ml 5 18 7,340
(dokter Dicky bilang mikroIU tapi di slidenya mIU, jadi
6 1,080 56,500
sebenarnya aku bingung hahaha), dan pada agglutination
test adalah 200 mIU/ml; sehingga sebenarnya strip test 78 7,650 229,000
lebih sensitive. Pada wanita menopause, kadar hCG sedikit 9 12 25,700 288,000
lebih tinggi, yakni 3,5-32 mIU/ml, sedingga terkadang 13 16 13,300 254,000
terdeteksi positif oleh strip test. Dengan demikian uji ini 17 24 4,060 165,400
adalah uji semiquantitaive, yakni kita dapat menentukan 25 40 3,640 117,000
minimal kadar hCG yang mungkin sudah disekresikan. Jika Non-pregnant females <5.0
mau yang kuantitatif, ya menggunakan sampel darah Postmenopausal females <9.5
menggunakan spektrofotometri (lebih mahal dan tidak
praktis).
Ovulasi merupakan pelepasan ovum dari ovarium yang PRAKTIKUM
terjadi 14 hari sebelum hari pertama menstruasi Prosedur:
berikutnya. Masa-masa sekitar sini adalah masa-masa Sebelum praktikum, satu orang probandus berkemih pada
paling fertile wanita, dan jika ada sperma yang berhasil gelas yang telah disediakan di kamar mandi.
menemui ovum di ampula tuba (day 3), maka ia akan Pertama, dilakukan dengan strip test.
tertarik secara kemotaksis ke cavum uteri (day 4) dan Buka strip test pack, lalu celupkan ke dalam sampel
memulai fase inisiasi, ada komunikasi dengan endometrium dari probandus sampai garis hitam selama tiga detik,
yang mau ditempelin (endometrium keluarin kayak angkat dan tunggu selama 5 menit.
pinopode yang bakal kayak jadi reseptornya embryo). Ini Buka strip test pack, lalu celupkan ke dalam kontrol
merupakan period window of implantation (sekitar 19-20 positif hamil sampai garis hitam selama tiga detik,
stelah HPMT atau 3-4 hari setelah fertilisasi). angkat dan tunggu selama 5 menit.
Setelah garis muncul, interpretasi.
Kedua, dilakukan dengan latex agglutination test 8. Apa sifat biologis dari LH? (ada yang jawab aTSH dan
Taruh 1 tetes sampel urin pada area reaksi pada slide bLH, ada yang jawab ahCG dan bhCG. Pendukung yang
menggunakan pipet bersih disposable. pertama mengatakan bahwa TSH, FSH, LH, dan hCG
Kocok reagen latex, lalu menggunakan droppernya memiliki subunit alpha yang sama, sedangkan LH
tetes 1 tetes ke sampel , lalu aduk dengan ujung punya subunit beta yang spesifik, oleh karena itu aTSH
pengaduk pipet. dan bLH paling benar. Pendukung yang kedua
Dengan pelan goyangkan slide selama 2 menit. Baca mengatakan bioaktivitas LH mirip hCG, sehingga
reaksi pada 2 menit.Harus dilihat dengan penerangan subunit ahCG dan bhCG lah yang paling benar. Saya
baik dan kalau perlu gunakan lup. Lihat ada tidaknya sendiri kurang tahu, hehehe, sekarang keputusan tetap
granulasi, interpretasikan. berada di tangan kalian)
The beta subunits vary. LH has a beta subunit of 120
Hasil dan Pembahasan: amino acids (LHB) that confers its specific biologic
Pada strip test, didapatkan: action and is responsible for the specificity of the
Sampel urin probandus : satu garis, yang interaction with the LH receptor. This beta subunit
menandakan dalam urin mengandung alpha hCG contains an amino acid sequence that exhibits large
tetapi tidak mengandung beta hCG (<25 homologies with that of the beta subunit of hCG and
mIU/ml) both stimulate the same receptor. However, the hCG
Sampel urin kontrol : dua garis, yang beta subunit contains an additional 24 amino acids,
menandakan dalam urin mengandung alpha hCG and the two hormones differ in the composition of
dan mengandung beta hCG. (>25 mIU/ml) their sugar moieties.(Jiang, 2014)
9. Kalo siklus 25 hari, maka ovulasi? (hari ke 11 HPMT)
10. Komponen hCG yang spesifik? Subunit bhCG

PRAKTIKUM PATOLOGI KLINIK: STRATEGY


DIAGNOSIS OF ANEMIA
Pemateri: Dinda | Editor: KD

Hai! Asik nih isinya gambar semua hehe, semangat guys jangan lupa doa
Pada agglutination test, didapatkan: sebelum dan sesudah belajar!
Sampel urin probandus : tidak ada granulasi, Friendly reminder : pahami dan hafalkan lagi materi praktikum A.6 yang
tidak mengandung beta hCG (<200 mIU/ml) ROUTINE BLOOD SCREENING FOR ANEMIA. Soalnya salah satu tujuan
Sampel urin kontrol : Ada granulasi, praktikum kita kali ini adalah mengingat kembali pemeriksaan:
hemoglobin, hematokrit, leukosit, trombosit, hitung jenis leukosit, dan
mengandung beta hCG (>200 mIU/ml) hitung retikulosit. Yuk sambil dibuka lagi HSC A.6nya
Biar nggak terlalu banyak aku langsung ke materi tambahannya aja yaa~

Melakukan Perhitungan Indeks Retikulosit


Jumlah retikulosit digunakan untuk
memperkirakan tingkat efektivitas eritropoiesis oleh
sumsum tulang, yang dapat dilaporkan sebagai jumlah
retikulosit absolut atau sebagai persentase retikulosit.
Kenapa seringkali persentase retikulosit saja tidak
cukup? Karena pada kondisi anemia, persentase
retikulosit nilainya akan kurang valid (palsu tinggi)
dan mungkin tidak mencerminkan respon sumsum
tulang yang sebenarnya.
Untuk itu, nilai persentase retikulosit perlu dikoreksi
berdasarkan hematokrit pasien dan kemudian
digunakan untuk menghitung IPR (Indeks Produksi
SOAL PRETEST POST TEST
Retikulosit) atau indeks retikulosit.
1. Apa itu ovulasi? (lepasnya ovum dr ovarium)
Indeks produksi retikulosit merupakan faktor yang
2. Ovulasi terjadi? (14 hari sebelum mens berikutnya)
disesuaikan berdasarkan dua hal yaitu derajat anemia
3. Setelah 20 hari HPMT, dalam urin ibu hamil dapat
dan waktu pematangan retikulosit.
ditemukan? (hCG)
4. Ovulasi terjadi karena? (LH Surge) Langkah-langkah perhitungan IPR :
5. Ngecek bhCG untuk? (Kehamilan trimester 1) 1. Menghitung persentase retikulosit =
6. Apa yang terjadi akibat menstruasi (ini aku lupa-lupaan
soalnya. Antara akibat atau menyebabkan)? Luteolisis
7. Hormon yang ningkatin suhu abis ovulasi? 2. Menghitung persentase retikulosit terkoreksi =
(Progesterone) persentase retikulosit x [hematokrit pasien/
hematokrit normal hematocrit (biasanya 45)]
3. Menghitung IPR = ANISOCYTOSIS
Ukuran eritrosis bervariasi.
Bisa dilihat dari RDW (Red
Nilai hematokrit Faktor koreksi* blood cell Distribution
45 1 Width) yang >14,5.
35 1,5 Berasosiasi dengan anemia
25 2 (terutama IDA,
15 2,5 megaloblastik dan
Setelah melakukan perhitungan retikulosit, maka hemolitik)
anemia dapat menjadi anemia hipoproliferatif
atau hiperproliferatif. DIMORFIK
Apabila faktor koreksi 2.5 maka perlu (Terdapat 2 variasi ukuran
pemeriksaan apusan darah tepi untuk memeriksa yang terlihat jelas,
morfologi sel-sel darah. Apabila pada apusan RDWnya juga >14,5.
ditemukan banyak sel berbentuk fragmen maka dengan transfusi, sindrom
dipikirkan terjadinya proses hemolitik. myelodysplastic, defisiensi
vitamin B12, defisiensi
Pemeriksaan Apusan Darah Tepi asam folat, defisiensi besi
Pada pemeriksaan apusan darah tepi, kita akan melihat (pada proses terapi).
1. Variasi ukuran dan warna eritrosit
2. Variasi bentuk dan distribusi
3. Adanya benda inklusi (inclusions in erythrocytes) VARIASI WARNA
VARIASI UKURAN Normalnya ukuran zona pucat di tengah berukuran
Eritrosit normal berbentuk bikonkaf dengan diameter sekitar 1/3 dari diameter eritrosit. Warna eritrosit
6 8 m, kira-kira sebesar inti dari limfosit kecil menggambarkan konsentrasi hemoglobin yang
(normal). Ukuran eritrosit dapat dilihat dari salah satu dikandung dan dapat dilihat salah satu indeks yaitu
indeks yaitu MCV MCHC yang dihitung dengan rumus :

HIPOKROMIA
Eritrosit lebih pucat dari
normal, walau bisa
NORMOSITIK
diperkirakan dengan MCHC,
Berasosiasi dengan
namun eritrosit hipokromik
perdarahan akut dan
tidak selalu MCHCnya
anemia karena penyakit
menurun. Berasosiasi
kronis.
dengan IDA, keracunan
timbal, dan beberapa kasus
MIKROSITIK anemia akibat penyakit
Berasosiasi dengan IDA kronis.
(Iron Deficiency Anemia), DIKROMIA
anemia sideroblastik, Terdapat 2 populasi
thalassemia minor, eritrosit dengan warna
keracunan timbal, berbeda. Kalau di contoh di
beberapa kasus inflamasi atas terlihat eritrosit
kronis dan normokromik dan
hemoglobinopati. hipokromik. Terjadi pada
anemia sideroblastik dan
transfusi.
MAKROSITIK
Berasosiasi dengan POLIKROMASIA
penyakit liver, defisiensi Terdapat RBC imatur/
vitamin B12, defisiensi retikulosit karena masih
asam folat, pada neonatus, ada RNA yang tertinggal,
dan retikulosis. membuat tampakan
keunguan). Berasosiasi
dengan perdarahan akut
dan kronis, pada neonatus,
dan pada anemia yang
terapinya efektif.
VARIASI BENTUK (POIKILOCYTOSIS) BADAN INKLUSI
ECHINOCYTES/BURR CELLS Normalnya tidak terdapat badan inklusi di dalam
RBC dengan penjuluran- eritrosit. Contoh badan inklusi :
penjuluran, biasanya dengan Polychromatophilic red cells (polikromasia) :
sentral pucat. Berasosiasi reticulocytes.
dengan uremia, defisiensi Protozoan inclusions : malaria (plasmodium)
pyruvate kinase, anemia
hemolitik mikroangiopati, DISTRIBUSI
pada neonatus (terutama
yang prematur), artefak. Bisa normal atau abnormal : rouleaux (kumpulan RBC
menempel rapi membentuk koin), aglutinasi (menyatu
secara irreguler)
SPHEROCYTES Bulat, lebih
gelap dan tidak punya area
pucat/central pallor). ALUR DIAGNOSIS ANEMIA (bagan ada dimodul)
Berasosiasi dengan Memahami alur diagnosis sangat penting untuk
hereditary spherocytosis, mengetahui jenis anemia dan selanjutnya menentukan
beberapa kasus anemia manajemennya. Sederhananya begini :
hemolitik, transfusi, dan 1. Lihat apakah 2 kondisi terpenuhi : Hb pria <13 g/dL
akibat luka bakar parah. atau wanita <12 g/dL DAN IPR 2,5
2. a. Jika YA : hitung indeks eritrosit (terutama MCV dan
MCHC)
SEL TARGET/CODOCYTES
normositik, normokromik curiga
Berwarna merah sampai
hipoproliferasi (penyakitnya liat sendiri di
merah salmon, di tengah
bagan ya) cek status besi (besi serum, TIBC,
RBC terlihat area
ferritin)
konsentrasi Hb yang gelap
jika TIBC normal/rendah, ferritin
dikelilingi oleh cincin perifer
normal/tinggi, dan besi serum normal/rendah
yang pucat. mirip mata
pertimbangkan pemeriksaan elektroforesis
banteng/bulls eye).
Hb (curiga hemoglobinopati) atau apusan
Berasosiasi dengan
darah maupun biopsi bone marrow jika tidak
hemoglobinopati,
ada penyakit kronis yang mendasari.
thalassemia, IDA,
mikrositik, hipokromik curiga defek
splenectomy, dan penyakit
maturasi cek status besi
liver obstruktif.
makrositik curiga defek maturasi cek
vitaminB12 dan asam folat
b. Jika TIDAK : lihat apusan darah tepi lihat
ELLIPTOCYTE (lonjong/cigar-
apakah banyak fragmented cells (RBC dengan
shaped) & OVALOCYTE
bentuk abnormal)
(bentuk oval/bulat telur).
Jika YA curiga hemolisis
Berasosiasi dengan hereditar
elliptocytosis/ovalocytosis, Jika TIDAK : perdarahan akut, atau
thalassemia mayor, IDA, pertimbangkan pemeriksaan lain seperti
anemia megaloblastik biopsi bone marrow
(macro-ovalocytes), anemia
myelophthisic. Bonus Nilai Normal :
Hb
Laki-laki : 13 18 g/dL
DACRYOCYTES/TEAR DROP Perempuan : 11,5 16 g/dL
CELLS Anak-anak : 11 16 g/dL
RBC berbentuk seperti Ibu hamil : 11 12 g/dL
tetesan air mata atau buah Hematokrit (Hct)
pir dengan tonjolan tumpul . Laki-laki : 40 48%
Berasosiasi dengan Perempuan : 37 43%
myelofibrosis primer, Hitung eritrosit
thalassemia, anemia Laki-laki : 4.2 6 x 1012/L
myelophtisic, dan gangguan Perempuan : 3.6 5.6 x 1012/L
hematopoiesis Neonatus : 5 6.5 x 1012/L
extramedullar lainnya. Indeks eritrosit
MCV : 80 100 fL
MCH : 27 31 pg
MCHC : 31 36 g/dL
Hitung retikulosit : 0.8 2% dari total eritrosit, atau 4. Subskripsi: arahan kepada apoteker, terdiri dari
<85 x 109/L. instruksi, jumlah, dan sediaan obat.
5. Signatura: petunjuk untuk pasien. Contoh: 3.d.d. Tab
TAMBAHAN DARI SOAL PRETEST 1 p.c. artinya minum 1 tablet 3 kali sehari setelah
1. Eritrosit normal berusia 120 hari. makan.
2. Whole blood yang disentrifugasi akan membentuk 3 6. Identitas pasien: nama, usia, berat badan (terutama
lapisan yaitu plasma, buffy coat (platelet dan WBC), anak-anak), tuliskan bila obat mengandung narkotika
dan packed RBC atau psikotropika
3. Hb x 10/ jumlah RBC adalah MCH (+) closing sign dan tanda tangan dokter
4. Anemia yang paling sering terjadi pada ibu hamil
adalah anemia defisiensi besi. Prosedur Praktikum (kemarin):
5. Anemia yang terjadi karena kekurangan faktor intrinsik Setiap mahasiswa menjawab pertanyaan yang tersedia (di
lambung adalah anemia pernisiosa (berarti defisiensi buku blok). Jawaban harus terdiri dari:
vitamin B12) 1. Regimen dosis (rute administrasi, frekuensi, dosis tiap
6. Anemia bisa dicek dari bone marrow (B) administrasi obat, waktu administrasi, dan durasi terapi)
7. Deteksi anemia yang lebih cepat dari MCV adalah cek 2. Sediaan obat yang sesuai dengan pasien
kadar Hb 3. Menghitung jumlah obat yang dibutuhkan
8. Jika Hb normal, TIBC tinggi, Fe serum dan ferritin
rendah, maka tergolong defisiensi besi latent.

Soal lain udah tercantum di materi yaa. Semoga


bermanfaat. SEMANGAT!!!

PRAKTIKUM FARMAKOLOGI: WRITING


PRESCRIPTION FOR PREGNANT AND LACTATING
WOMEN
Pemateri: Rasya | Editor: Yudha
Bismillahirrahmanirrahim, langsung dimulai aja ya
Btw, ini yang penulisan resep aku taruh atas aja ya -ed

Lanjut ke bagian yang penting ya!!!!!!!!!!!!!

PERUBAHAN FARMAKOKINETIK PADA KEHAMILAN


+ Absorpsi:
Motilitas gastrointestinal dapat berkurang saat kehamilan
sehingga mengakibatkan absorpsi obat yang diberikan
PRINSIP PENULISAN RESEP melalui oral melambat. Mual, muntah, dan pengosongan
1. Identitas dokter atau penulis resep: nama, nomor surat lambung yang lambat dapat mengubah absorpsi obat.
ijin praktik (SIP), alamat Peningkatan pH lambung akibat kehamilan dapat
2. Superskripsi: simbol R/, tempat dan tanggal resep mempengaruhi absorpsi asam lemah dan basa.
ditulis
3. Inskripsi: obat yang diresepkan dengan nama dan
jumlah atau kekuatan tiap bahan obat
+ Distribusi: EFEK OBAT TERHADAP KEHAMILAN, JANIN, DAN
Konten lemak dan air maternal akan meningkat selama NEONATUS
kehamilan dan dapat meningkatkan volume distribusi + Teratogenisitas:
obat. Distribusi volume obat larut emak juga akan naik. Paparan obat pada trimester pertama berisiko tinggi
Sebaliknya, konsentrasi albumin di plasma akan menyebabkan malformasi dan ideanya semua terapi
berkurang sehingga menyebabkan peningkatan distribusi harus dihentikan sebelum melakukan konsepsi. Paparan
obat yang berikatan kuat dengan protein. Namun, obat- obat yang tidak disengaja sering terjadi karena setidaknya
obat yang tidak berikatan dengan protein lebih cepat setengah dari kehamilan merupakan kehamilan yang
dibebaskan oleh hati dan ginjal saat kehamilan sehingga tidak direncanakan. Malformasi diperikana
konsentrasinya hanya sedikit berubah. Namun, masalah mempengaruhi 2-4% dari semua kelahiran hidup. Pada
bisa muncul bila konsentrasi obat digunakan untuk sebagian besar kasus, penyebab abnormalitas tidak
menyesuaikan terapi obat seperti antikonvulsan. diketahui. Faktor eksogenik seperti obat diperkirakan
Pengukuran rutin konsentrasi obat dilakukan untuk menyebabkan 1-5% malformasi (mempengaruhi 0,2% dari
mengukur konsentrasi total seperti obat terikat (bound) total kelahiran hidup). Persentase kehamilan yang
dengan obat bebas (unbound). Total konsentrasi obat dipengaruhi obat kecil namun dapat dicegah. Sulit untuk
dapat menurun saat kehamilan, sehingga untuk obat-obat memprediksi kehamilan (yang terekspos teratogen) mana
seperti fenitoin, pengukuran konsentrasi obat unbound yang dapat mengakibatkan malformasi.
penting dilakukan sebelum dan selama kehamilan. Obat-obat teratogenik: ACE inhibitor, androgens,
antineoplastics (beberapa), carbamazepine, carbimazole,
+ Metabolisme dan Eliminasi: danazol, diethylstilbestrol, lithium, misoprostol,
Selama kehamilan, volume plasma maternal, cardiac penicillamine, phenytoin, tetracyclines, thalidomide,
output, dan filtrasi glomerular naik dari 30%-50%, valproic acid, vitamin A & derivatnya seperti isotretinoin,
berpotensial menurunkan konsentrasi obat-obat yang warfarin
dibebaskan oleh ginjal. Artinya, kenaikan dosis obat yang Waktu paparan obat yang tepat dapat membantu penilaian
metabolized dan dieliminasi oleh ginjal dibutuhkan saat risiko janin karena sebagian obat hanya menyebabkan
kehamilan. Contohnya, oabt seperti penicillin akan lebih kelainan sepsifik pada periode waktu tertentu saat
cepat dibebaskan karena kenaikan filtrasi glomerular. kehamilan. Misalnya, antagonis asam folat
Kadar estrogen dan progesteron tinggi dapat mengubah (carbamazepine) tidak menyebabkan defek pada neural
aktivitas enzim di hati dan meningkatkan eliminasi tube bila paparan terjadi 4 minggu setelah konsepsi, di
sebagian obat namun dapat menyebabkan akumulasi mana sudah terjadi penutupan neural tube, tetapi masih
obat lainnya. Peningkatan metabolisme obat di hati dapat bisa menyebabkan malformasi lainnya.
bervariasi namun dapat menyebabkan peningkatan
kebutuhan obat pada trimester ketiga seperti fenitoin dan + Risiko Farmakologis:
methadone. Obat Efek samping
ACE inhibitor Disfungsi renal, oligohidramnion,
TRANSFER OBAT TRANSPLASENTAL intrauterine frowth retardation (IUGR)
Laju perpindahan obat dapat dipengaruhi berbagai faktor Antidepresan Withdrawal reactions
seperti: Antihipertensif Hipoksia fetal dengan treatment
Berat molekul: obat-obat dengan berat molekul < 500 Da berlebih karena perfusi plasenta
dapat menembus plasenta dengan mudah, sedangkan menurun
obat-obat dengan berat molekul lebih besar (600 1000 Kortikosteroid Adrenal supression
Da) menembus plasenta lebih lambat. Obat-obat dengan NSAID Premature ductus arteriosus closure,
berat molekul > 1000 Da seperti insulin dan heparin tidak renal impairment
dapat menembus plasenta dalam jumlah yang signifikan. Opioid Withdrawal reactions
Kelarutan lipid: obat-obat lipofilik seperti opiates dan
antibiotik lebih mudah menembus plasenta dibandingkan + Kategori Risiko Penggunaan Obat saat Kehamilan
obat-obat larut air. menurut U.S. FDA:
Derajat ikatan dengan protein: albumin plasma maternal Kategori A: Pada studi kontrol terhadap wanita tidak
berkurang sedangkan albumin plasma fetal meningkat ditemukan adanya risiko terhadap janin dan sedikit
saat kehamilan sehingga meningkatkan konsentrasi obat- kemungkinan bahaya pada janin.
obat yang berikatan dengan protein pada fetus/janin. Kategori B: Studi terhadap hewan coba menunjukkan
Muatan listrik: pH janin sedikit lebih asam daripada pH tidak ada risiko pada janin namun tidak ada studi yang
ibu sehingga basa lemah lebih mudah menembus dilakukan terhadap manusia atau studi pada hewan coba
plasenta. Ketika sudah masuk sirkulasi fetal, molekul basa menunjukkan adanya efek samping namun tidak ada pada
lemah tersebut akan terionisasi dan cenderung tidak studi yang dilakukan terhadap wanita hamil.
kembali ke sirkulasi maternal. Kategori C: Studi pada hewan coba menunjukkan efek
samping pada janin dan tidak ada studi kontrol terhadap
manusia atau tidak ada studi yang dilakukan terhadap
manusia maupun hewan.
Kategori D: Ada bukti positif risiko terhadap janin namun
keuntungan dari penggunaan obat tersebut terhadap ibu
hamil dapat diterima (contoh: obat tersebut dibutuhkan kehamilan manusia. Sebagian besar antasida (kecuali
dalam kondisi yang mengancam nyawa atau penyakit natrium bikarbonat) aman digunakan selama kehamilan
serius di mana obat lain tidak bisa digunakan atau tidak dan untuk aspirasi prokfilasis atau saat persalinan
efektif) karena absorpsi yang minimal. Namun, magnesium
Kategori X: studi pada hewan maupun manusia trisilikat tidak boleh digunakan pada dosis tingfgi dan
menunjukkan adanya kelainan pada janin atau ada bukti jangka panjang pada wanita hamil.
risiko dari manusia (atau keduanya), atau risiko Breastfeeding
penggunaan obat pada ibu hamil lebih besar daripada Tidak ada data ekskresi Aluminum hydroxide /
keuntungan dari obat. Obat tersebut menjadi magnesium hydroxide / simethicone di dalam ASI.
kontraindikasi terhadap wanita yang akan atau sedang Sucralfate
hamil. Generik:
Tablet 500 mg
OBAT-OBAT YANG DIGUNAKAN SAAT KEHAMILAN, Dosis oral:1 gr secara
Suspensi 500 mg/5 ml
PERSALINAN, DAN MENYUSUI oral ke lambung kosong
Nama dagang: Inpepsa
+ Suplemen dan Mineral: 4 kali sehari.
Suspensi 500 mg/mL (botol
Dosage Pharmaceutical drug form 100 ml dan 200 ml)
Ferrous Pregnancy
Dosis oral: Sucralfate dimasukkan ke dalam kategori B oleh FDA.
Generik:
Suplemen untuk wanita
Ferrous sulfate film Breastfeeding
hamil:60 mg elemental Tidak ada data ekskresi sucralfate pada ASI.
coated tablet 300 mg
ferrous Sulfate ferrous
Sirup 15 mg/5 ml Ranitidine
300 mg per hari
Dosis oral: 150 mg 2 kali
Asam folat
sehari atau 300 mg Generik:
1. Generik:Tablet 400 g, 1 sebelum tidur selama 8 Tablet 150 mg
Dosis oral: mg dan 5 mg minggu atau 12 minggu Injeksi 25 mg/ml
Suplemen untuk wanita 2. Nama dagang bila dibutuhkan. Durasi Nama merk: Rantin
hamil: 400 g per hari Folavit:Tablet 400 g dan 1 treatment GERD: 2 Tablet 150 mg dan 300 mg
mg minggu
Pregnancy
+ Obat-obat untuk Gastrointestinal: Ranitidine dimsasukkan ke dalam kategori kehamilan B
=====GERD===== oleh FDA.
Dosage Pharmaceutical drug form Breastfeeding
Antacida (Aluminium hydroxide dan magnesium Ranitidine diekskresikan ke dalam ASI dalam konsentrasi
hydroxide) melebihi ranitidine di plasma.
Generik: Cimetidine
Tablet kunyah: Aluminium Dosis oral: 800 mg 2 kali
Generik:
hidroksida 200 mg & sehari atau 400 mg 4
Tablet 200 mg
Magnesium Hidroksida kali sehari. Durasi
Nama merk: Corsamet
Dosis oral:Aluminium 200 mg treatment GERD: 12
Tablet 200 mg dan 400 mg
hydroxide / Magnesium Suspensi per 5 ml minggu
hydroxide = 200 mg - (Aluminium hidroksida 200 Pregnancy
200 mg hingga 800 mg - mg & Magnesium Cimetidine dimasukkan ke dalam kategori B oleh FDA.
800 mg 4 kali sehari. Hidroksida 200 mg) Cimetidine aman digunakan saat kehamilan untuk
mengobati peptic ulcer disease berat.
Dosis harian maksimum Nama dagang: Mylanta Breastfeeding
Aluminium hydroxide / (Aluminium hydroxide Cimetidine diekskresikan ke dalam ASI.
Magnesium hydroxide: danmagnesium hydroxide, Omeprazole
4800 mg - 4800 mg 4 dan simethicone) GERD simptomatik
kali sehari. Jangan Tablet kunyah: Al tanpa lesi esofageal:
menggunakan dosis hidroksida 200 mg, Mg Oral: 20 mg sehari Generik:
harian maksimum kebih Hidroksida 200 mg dan selama 4 minggu. Kapsul 20 mg
dari 2 minggu. Simethicone 20 mg Injeksi 40 mg/10 ml
Suspensi per 5 ml (Al GERD simptomatik Nama dagang: Conprazole
hidroksida 200 mg & Mg dengan gejala Kapsul 20 mg
Hidroksida 200 mg) dalam tambahan (contoh Nama dagang: Gastrofer
botol 150 ml atau 360 ml erosive esophagitis): Injeksi 40 mg/10 ml
Pregnancy Oral: 20 mg sehari
Aluminum hydroxide / magnesium hydroxide / selama 4-8 minggu.
simethicone belum dimasukkan ke dalam kategori Pregnancy
manapun oleh FDA. Belum ada data kontrol pada Omeprazole dimasukkan ke dalam kategori C oleh FDA.
Breastfeeding Pregnancy
Konsentrasi omeprazole dapat diukur di dalam ASI. Metoclopramide dimasukkan ke dalam kategori B oleh
Karena berpotensi menimbulkan efek samping serius FDA. Metoclopramide digunakan saat kehamilan karena
dan kemungkinan tumorigenisitas, obat ini tidak boleh terdapat efek antiemesis dan gastric prokinetic.
digunakan selama periode laktasi.
=====Konstipasi=====
=====Mual dan Muntah===== Dosage Pharmaceutical drug form
Dosage Pharmaceutical drug form Lactulose
Pyridoxine
Generik:
Generik:
Oral: 10 mg/dosis, 4 kali Oral: 10-25 ml/dosis sir 3,335 g/5 mL
Tablet 10 mg dan 25 mg sesuai kebutuhan, 3 kali
sehari Nama dagang: Lactulax
Injeksi 100 mg/ml sehari. Durasi Lactulaxsyr 3.335 g/5 mL
Pregnancy tresatment 1-3 hari (60 ml, 120 ml, and 200
Pyridoxine dimasukkan ke dalam kategori A. Kebutuhan ml)
pyridixone meningkat saat kehamilan.
Pregnancy
Breastfeeding
Lactulose dimasukkan ke dalam kategori B dan hanya
Kebutuhan pyridoxine meningkat saat menyusui. Tidak
digunakan bila benar-benar dibutuhkan.
diketahui apakah obat ini diekskresikan ke dalam ASI. Breastfeeding
Doxylamine Tidak diketahui apakah obat ini diekskresikan ke dalam
Oral: 10 mg 1x sehari, ASI atau tidak.
taken at night Di Indonesia, sediaan obat Bisacodyl
Doxylaminein doxylamine tidak tersedia Generik:
dikombinasikan dengan dalam komponen tunggal. Enteric coated tablet 5 mg
pyridoxine
Oral: 5-15 mg/dosis, Suppositoria 5 mg dan 10
Pregnancy sesuai kebutuhan, 1 kali mg
Doxylamine dimasukkan ke dalam kategori A.
sehari. Durasi treatment Nama merk: Dulcolax
Breastfeeding
1-3 hari Enteric coated tablet 5 mg
Doxylamine succinate tidak boleh digunakan oleh
Suppositoria 5 mg dan 10
wanita menyusui.
mg
Dimenhydrinat
Pregnancy
Dosis untuk mual dan
Bisacodyl dimasukkan ke dalam kategori B dan hanya
muntah (bila pyridoxine
digunakan bila benar-benar dibutuhkan.
dan doxylamine tidak
Breastfeeding
cukup): Generik:
Tidak diketahui apakah obat ini dieksresikan ke dalam
Oral atau suppositoria: Tablet 50 mg
ASI atau tidak.
50-100 mg/dosis, 4-6 kali Nama dagang: Dramamin
sehari (max 200 mg/hari Tablet 50 mg
+ Obat-Obat untuk Infeksi
bila dimakan dengan 4
Dosage Pharmaceutical drug form
tablet doxylamin /
piridoksin) Drug Classification: PENICILLIN
Pregnancy 1. Amoxicillin
Dimenhydrinat dimasukkan ke dalam kategori B. Dosis oral untuk
Breastfeeding uncomplicated Generik:
Dimenhydrinate diekskresikan ke dalam aSI dalam pneumonia: Kapsul 250 mg; 500 mg
jumlah yang sedikit. Dewasa: 250-500 mg Dry syr. 125 mg/5ml (60
Metoclopramide /dosis setiap 8 jam ml/flask)
Generik: Durasi treatment: 7-10 Dry syr. Forte 250
Tablet 5 mg dan 10 mg hari
mg/5ml (60 ml/flask)
Sirup 5 mg/5 ml Nama merk: Amoxan
Gonorrhea Acute,
Injeksi 5 mg/ml Kapsul 250 mg; 500 mg
uncomplicated ano-
genital dan urethral Dry syr. 125 mg/5ml (60
5-10 mg/dosis, setiap 8 Nama dagang: Primperan
jam Tablet 5 mg dan 10 mg infections ml/flask)
Sirup 5 mg/5ml Dosis: amoxicillin 3 g Dry syr. Forte 250
PrimperanPaed oral drops oral + probenecid 1 g mg/5ml (60 ml/flask)
1 mg/10 tetes oral dosis tunggal
Injeksi 10 mg/2 ml (Cavenee et al., 1993)
Pregnancy
Amoxicillin dimasukkan ke dalam kategori B oleh FDA.
Breastfeeding
Amoxicillin diekskresikan ke dalam ASI dalam jumlah
yang sedikit. 2. Ceftriaxone
2. Procainebenzylpenicillin/Procaine penicillin G Dosis untuk Pneumonia
Generik: 1 - 2 g IV atau IM setiap 24
Dry injection jam selama 7-21 hari,
1.000.000 UI/ vial tergantung penyebab dan
(intramuscular) keparahan infeksi
Early syphilis (stage 1-2):
Dry injection Generik:
Procaine benzylpenicillin, Dosis N. gonorrhoeae Dry inj 1g/vial
3.000.000 UI/ vial
1.2 juta IU sehari, injeksi (direkomendasikan oleh Brand name: Ceftricor
(intramuscular)
intramuskuler, 10 hari CDC untuk wanita Dry inj 1g/vial
Nama merk: Procain
berturut-turut.
Penicillin-G Meiji hamil):dual therapy terdiri
Dry injection dari ceftriaxone 250 mg
3.000.000 UI/ vial dalam dosis IM tunggal dan
(intramuscular) azithromycin 1 g oral
Pregnancy sebagai dosis tunggal.
Procaine benzylpenicillin dimasukkan ke dalam kategori Pregnancy
B oleh FDA Ceftriaxon dimasukkan ke dalam kategori B oleh FDA.
Breastfeeding Breastfeeding
Penicillin diekskresikan melalui ASI. Konsentrasi ceftriaxone rendah diekskresikan di dalam
3. Benzathinebenzylpenicillin / Benzathine penicillin G ASI.
/ Benzylpenicillinbenzathine / Benzathine penicillin
Generik: Drug Classification: MACROLIDE
Sifilis awal (stadium 1-2):
Dry injection 1. Erythromycin
Benzathinebenzylpenicillin
1.200.000 UI/vial Dosis untuk infeksi
2,4 juta IU, injeksi
(intramuscular) Chlamydia
intramuscular, pada sesi
Dry injection -Infeksi ringan-sedang: 250- Generik:
tunggal. (2 injeksi di tempat
2.400.000 UI/vial 500 mg (base, estolate, Kapsul 250 mg
yang berbeda)
(intramuscular) stearate) atau 400-800 mg Tablet 500 mg
Pregnancy: Benzathinebenzylpenicillin dimasukkan ke (ethylsuccinate) oral setiap Dry syr 200 mg/5ml
dalam kategori B oleh FDA 6 jam (60 ml/flask)
Breastfeeding -Infeksi berat: 1-4 g/hari IV Nama merk: Erysanbe
Penicillin G diekskresikan ke dalam ASI dan tidak ada pada dosis berbeda setiap 6 Kapsul 250 mg
risiko terhadap bayi. jam atau dengan infus. Tablet kunyah 200 mg
Dry syr 200 mg/5ml
Drug Classification: CEPHALOSPORINS Dosis alternatif untuk Early (60 ml/flask)
1. Cefadroxil syphilis (stadium 1-2):Oral FC tablet 500 mg
Generik: 500 mg setiap 6 jam selama
30 hari.
Kapsul 250 mg; 500
Dosis untuk Upper mg Pregnancy
Respiratory Tract Infection Dry syr. 125mg/5ml Erythromycin dimasukkan ke dalam kategori B oleh
500 mg secara oral setiap 12 FDA. Erythromycin melewati plasenta dalam jumlah
(60 ml/flask)
jam atau or 1 g secara oral Dry yang sedikit.
syr. Forte
setiap 24 jam selama 7-10 Breastfeeding
250mg/5ml (60
hari. Erythromycin diekskresikan ke dalam ASI dalam jumlah
ml/flask) yang sedikit.
Nama merk: Cefat 2. Azithromycin
Dosis untuk Urinary Tract
Kapsul 250 mg; 500 Generik:
Infection
mg Dosis untuk N.
-Uncomplicated: 1 to 2 Tablet 250 mg dan 500
g/hari secara oral in 1 Dry syr. 125 mg /5ml gonorrhoeae (rekomendasi
mg
(60 ml/flask) CDC untuk wanita
hingga 2 dosis terpisah Dry syr 200 mg/5ml (60
-Complicated: 1 g secara Dry syr. Forte 250 mg / hamil):dual therapy
ml/flask)
oral 2 kali sehari 5ml(60 ml/flask) terdiri dari ceftriaxone 250
Nama merk: Aziwin
mg dalam dosis IM tunggal
dan azithromycin 1 g oral Tablet 250 mg dan 500
sebagai dosis tunggal. mg
Pregnancy
Cefadroxil dimasukkan ke dalam kategori B oleh FDA.
Pregnancy
Breastfeeding
Azithromycin dimasukkan ke dalam kategori B oleh
Pada 1 gram dosis oral terhadap 6 ibu menyusui, kadar
FDA. Azithromycin ditolerir lebih baik daripada
cefradoxil mencapai puncak dalam 6-7 jam dengan
erythromycin.
kadar rata-rata 1,83 mcg/ml. Kadarnya cukup rendah
Breastfeeding
dan tidak menimbulkan efek samping terhadap bayi.
Azithromycin diekskresikan ke dalam ASI. samping yang dapat muncul pada bayi antarar lain
3. Spiramycin refusal of the breast, intestinal gas, and muntah berat.
Dosis untuk infeksi Selain itu, terdapat risiko supresi sumsum tulang and
Toxoplasma gondii selama Gray Syndrome pada bayi.
kehamilan: 1 g (3 juta U) Drug Classification: SULFONAMIDES AND
setiap 8 jam (total 3 g atau TRIMETHOPRIM
9 million U per hari) Obat ini digunakan untuk mengobati UTI namun
merupakan kontraindikasi bagi wanita hamil dan
Terapi spiramycin harus Generik: menyusui.
dilanjutkan hingga Tablet 250 mg dan 500 Pregnancy
persalinan pada wanita mg Sulfamethoxazole-Trimetrophrime (cotrimoxazole)
dengan kecurigaan infeksi Nama merk: Ismacrol digolongkan ke dalam kategori D oleh FDA. Pada tikus,
rendah atau dengan hasil Tablet 500 mg dosis oral 533 mg/kg atau 200 mg/kg menghasilkan efek
amniotic fluid PCR negatif teratologik dengan manifestasi cleft palates. Belum ada
saat follow-up Spiramycin data yang ditemukan pada manusia
tidak bisa digunakan untuk Breastfeeding
mengobati janin yang telah Kadar sulfamethoxazole-trimethoprim di dalam ASI
terinfeksi Toxoplasma sekitar 2 - 5% dari dosis yang direkomendasikan untuk
gondii. bayi berusia > 2 bulan. Sulfamethoxazole-trimethoprim
Pregnancy dikontraindikasikan untuk ibu menyusui.
Tidak teratogenik, tidak dapat menyembuhkan infeksi Drug Classification: AMINOGLYCOSIDE
pada janin; diindikasikan kepada wanit hamil yang Nama obat: streptomycin, gentamicin, kanamycin,
dicurigai terinfeksi toxoplasma selama masa gestasi 18 tobramycin, amikacin, dll. Streptomycin digunakan
minggu. untuk mengobati infeksi Tuberculosis. Gentamycin and
amikacin digunakan untuk mengobati infeksi bakteri
Drug Classification: IMIDAZOLE DERIVATES gram negatif. Namun, obat-oba tersebut adalah
Metronidazole kontraindikasi bagi ibu hamil.
Dosis untuk Bacterial Pregnancy
Vaginosis Aminoglycoside digolongkan ke dalam kategori D oleh
Nonpregnant women: FDA. Aminoglycoside dapat menembus plasenta. Belum
Generik:
-Immediate-release ada data dari studi kehamilan pada manusia; namun
Tablet 250 mg dan 500
tablets (recommended ditemukan ada kerusakan pada nervus cranial VIII pada
mg
regimen): 500 mg oral Ovula 500 mg janin dengan ketulian bilateral. Salah satu dari
setiap 12 jam untuk 7 hari aminoglycoside, gentamycin, dapat menyebabkan
Syr 125 mg/5ml (60 ml/ nephrotoksisitas in pada janin tikus.
-Extended-release tablets
flask) Breastfeeding
(alternative regimen): 750
Infus 5 mg/ml Aminoglycoside diekskresikan ke dalam ASI. Karena
mg oral sekali sehari untuk
Brand name: Flagyl bioavailabilitas oral rendah, kemungkinan tidak ada
7 hari
Tablet 500 mg keracunan sistemik yang dapat terjadi pada bayi.
Pregnant women: Ovula 500 mg dan 1000 Drug Classification: FLUOROQUINOLONE
Immediate-release tablets: mg Nama obat: levofloxacine, ofloxacine, ciprofloxacin, dl.
500 mg oral 2 kali sehari Suspensi 125 mg/5ml Fluoroquinolons digunakan untuk mengobati UTI,
untuk 7 hari atau 250 mg (60 ml/flask) lower respiratory tract infection, pneumonia, anthrax,
oral 3 kali sehari untuk 7 dll tetapi merupakan kontraindikasi pada wanita hamil
hari Pregnancy
Pregnancy Kategori FDA C. Secara umum, tidak boleh digunakan
Metonidazole dimasukkan ke dalam kategori B oleh saat kehamilan. Dosis tinggi terhadap hewan coba tidak
FDA. menyebabkan keracunan, namun ada keracunan pada
Metronidazole terdapat di dalam ASI dengan kadar janin (fetotoxicity). Kerusakan kartilago and
yang sama dengan kadar serum ibu dan kadar serum arthropathy ditemukan dan dapat mempengaruhi
bayi bisa sama dengan kadar bayi terapetik. pertumbuhan tulang pada janin. Karena banyak
alternatif yang lebih aman, beberapa ahli mengatakan
bahwa fluoroquinolone dikontraindikasikan saat
Drug Classification: AMPHENICOL
kehamilan terutama pada trimester pertama.
Pregnancy
Breastfeeding
Chloramphenicol and digolongkan ke dalam kategori C
Quinolone-induced cartilage erosion and arthropathies
oleh FDA. Chloramphenicol oral dapat menembus
ditemukan pada hewan-hewan muda sehingga bisa
plasenta. Penggunaan obat ini pada usia kehamilan tua
memberikan efek toksik terhadap pertumbuhan sendi
berhubungan dengan efek samping terhadap bayi
pada bayi.
seperti gray baby syndrome.
Breastfeeding
+ Obat-Obat Tokolitik
Chloramphenicol diekskresikan ke dalam ASI. Efek
Drug Classification: DIHYDROPIRIDINE CLASS OF dalam 1000 ml NaCl 0,9%/Ringer Laktat, berikan infus
CALCIUM CHANNEL BLOCKERS dengan kecepatan 40 tetes/menit hingga perdarahan
Nifedipine berhenti.
Tocolytic hanya bisa
diberikan hingga 48 hours Magnesium Sulfate
setelah muncul gejala Magnesium sulfate digunakan untuk mengobati eclampsia
kelahiran premature. (level of competence 3B).
Dosis yang digunakan Generik: Dosis: dosis awal 4-gram bolus iv dalam10 ml (MgSO4 40%)
pada studi yang dilakukan Kapsul 10 mg infus selama 20 menit. Bila rute IV sulit dilakukan berikan
berada pada rentang 10- Nama merk: Adalat injeksi IM pada m. glueus maximus dextra & sinistra. Tiap
40 mg sebagai dosis awal Tablet 5 dan 10 mg injeksi mengandung 5 g MgSO4 (12,5 ml MgSO4 solution
pertama. Dosis lanjutan 40%) IM
berada pada rentang 10- Generik: Injeksi 20%, Injeksi 40%
20 mg setiap 6-8 jam bila
perlu. Pregnancy Warnings
Pregnancy Magnesium sulfate dimasukkan ke dalam kategori D oleh
Nifedipine dimasukkan ke dalam kategori C oleh FDA. FDA.
Namun, nifedipine digunakan dengan aman oleh Breastfeeding Warnings
manusia untuk terapi preeclampsia dan kelahiran Magnesium diekskresikan ke dalam ASI.
prematur.
Breastfeeding
Nifedipine diekskresikan ke dalam ASI.

Drug Classification: 2 AGONIST


TERBUTALINE
Untuk persalinan
premature:
Infus IV: larutkan 1000 g
(2 ampul) ke dalam infus
NaCl 0,9% 500 ml. Dosis Generik:
awal 20 g dan dapat naik Tablet 2,5 mg
10 g selama 15 menit
Syrup 1,5 mg/5 ml
hingga persalinan. Dosis
Inj 0,5 mg/ml
maksimum 80 mcg/ menit.
Nama merk: Bricasma
Tablets: 2.5-7.5 mg oral
Tablet 2,5 mg
setiap 6 jam. Terapi harus
dilanjutkan hingga 36-37 Infus Bricasma 0.5
minggu gestasi. mg/mL
Injeksi subkutan: 0.25 mg
setiap 6 jam, harus
dilanjutkan hingga
persalinan.
Pregnancy
Terbutaline dimasukkan ke dalam kategori B oleh FDA.
Namun, terbutaline dapat menembus plasenta sehingga
janin dapat menerima efek samping.
Breastfeeding
Terbutaline diekskresikan ke dalam ASI.

+ Obat-Obat yang Digunakan saat Persalinan


Oxytocin
Sediaan obat: Injeksi 10 unit/ml dalam ampul 1 ml
Pada tahap ketiga persalinan normal (Kompetensi 4A):
Oxytocin 10 unit intramuscular dosis tunggal. Berikan
segera setelah kelahiran janin dan plasenta.
Bila terjadi perdarahan post-partum (Kompetensi 3B)
terjadi karena atonia uterior retensio placenta
Treatment: Oxytocin 20-40 unit, dilarutkan ke dalam 1000
ml NaCl 0,9%/Ringer Laktat, berikan infus dengan
kecepatan 60 tetes/menit. Berikan oksitosin 10 unit secara
intramuscular. Lanjutkan dengan Oxytocin 20 unit, larutkan

You might also like